Naked Science Forum

Non Life Sciences => Physics, Astronomy & Cosmology => Topic started by: Bogie_smiles on 11/06/2018 17:26:12

Title: What is Nothingness?
Post by: Bogie_smiles on 11/06/2018 17:26:12

Opening Post:
What is nothingness?


This is a topic that comes up to help differentiate between various cosmological models. It is a topic that is discussed in regard to the explanation of the existence of the universe. Case in point, Big Bang Theory, the consensus cosmology; though BBT doesn’t address the beginning directly, it leads to discussions of a Singularity referred to as simply the Big Bang event.

One option, mentioned only in passing in this OP (not intended for discussion, but that is commonly preferred in the religious community), is that God created the universe. That isn’t considered a scientific explanation, and I’m not entertaining any discussion about the Supernatural in this thread. The Supernatural is excluded from scientific discussions on the basis that the scientific method doesn’t recognize the Supernatural.

A second explanation, and the one under which the Big Bang Singularity falls, is that the universe spontaneously was generated out of nothingness, and several proposals that might apply to that explanation are mentioned: The Singularity, Quantum fluctuations, spontaneous symmetry breaking are a few.

The third explanation is that the universe has always existed. There are various models, like the steady state models, cyclical models, and the idea referred to as the Perfect Cosmological Principle, which states that the universe is homogeneous and isotropic on a grand scale in both space and time. That view says the universe looks the same everywhere (on the large scale), the same as it always has and always will (a steady state).

One definition of nothingness:
No space, no time, no energy, and no potential for any space, time or energy.

Does that definition of nothingness seem complete? If not, how would you change it to better express the concept of nothingness in regard to cosmology? What cosmological models address something from nothingness, and what explanation is given for such an event.
Title: Re: What is Nothingness?
Post by: Bill S on 11/06/2018 19:48:08
Quote
What cosmological models address something from nothingness, and what explanation is given for such an event.

The first part might not present too great a problem; but I genuinely wish you luck with the second part.

I'll be quite happy to chip in later, but I think there is a tendency to run away when I climb on my soapbox about "something from nothing", so I'll hold off rather than put your chances at risk.
Title: Re: What is Nothingness?
Post by: Colin2B on 12/06/2018 08:10:16
Big Bang Theory, the consensus cosmology; though BBT doesn’t address the beginning directly, it leads to discussions of a Singularity referred to as simply the Big Bang event.
The original BBT had a point singularity, but most modern theories are still called BBT and don’t. Current lead view, still being debated and tested, is inflation but it (and most of the others) don’t go right back to a point of nothingness. Remember, singularities aren’t nothing; and quantum fluctuations - a vacuum where particles pop in and out of existence aren’t nothing - except for a infinitesimally short time.
There is a strong view from qm that ‘nothing’ is inherently unstable and if it did exist it was for a very, very short time.
So, most theories avoid the question by not dealing with nothing at all.
Title: Re: What is Nothingness?
Post by: Bogie_smiles on 12/06/2018 15:00:47
Reply #3


The original BBT had a point singularity, but most modern theories are still called BBT and don’t. Current lead view, still being debated and tested, is inflation but it (and most of the others) don’t go right back to a point of nothingness. Remember, singularities aren’t nothing; and quantum fluctuations - a vacuum where particles pop in and out of existence aren’t nothing - except for a infinitesimally short time.
There is a strong view from qm that ‘nothing’ is inherently unstable and if it did exist it was for a very, very short time.
So, most theories avoid the question by not dealing with nothing at all.
In order for a cosmological model to fit in the second category, “Something from nothingness”, it would seem necessary that they address the issue of a beginning. Your point, that the condition of nothingness is [perhaps intentionally] avoided, is well taken. It is probably true because there is no scientific explanation for how the universe could exist, if at first there was nothingness.

It is fair to correct the OP to say that the models that don’t address the issue directly do not necessarily intended to suggest the universe emerged from nothingness. It is also appropriate to correct the OP to acknowledge the more recent current lead view of BBT to include Inflationary Theory, and to recognize the more current contending models.

Those improvements to the OP make category two, something from nothing, a lonely place.

Is it fair to say that those models that avoid addressing the beginning, but that don’t intend to convey the idea that they are “Something from nothing” models, might actually fit into the category of “Always existed” when more knowledge about preconditions become available?

Title: Re: What is Nothingness?
Post by: Bill S on 12/06/2018 15:48:23
Quote from: Colin
Remember, singularities aren’t nothing; and quantum fluctuations - a vacuum where particles pop in and out of existence aren’t nothing - except for a infinitesimally short time.

I’m fine with the first part of that, but wonder about the idea that something can be nothing, even for an “infinitesimally short time”, and can then become something again.

Perhaps it makes a difference if we are distinguishing between actual particles and virtual particles.

My understanding is that it is virtual particles that pop in and out of existence; and that Matt Strassler advises not thinking of them as particles.
 
Be that as it may, these virtual particles borrow energy from the vacuum for such a short time that it the vacuum “doesn’t notice”.  Putting it that way leaves me with the question: if the “particle” was nothing before it popped into existence, what borrowed the energy?

I’ll try a different approach.  Let’s go for not thinking of the “emergence” of the particle as its origin. 
The vacuum is a mass of virtual particles constantly popping in and out of existence.  These are not different particles, constantly being created and destroyed; rather they exist as the vacuum energy that is always present.  If this interpretation were correct, every particle would exist, in some form, at all times. It would never be “nothing”, even for an “infinitesimally short time”.

I tried to stay out - honest Guv!
Title: Re: What is Nothingness?
Post by: Colin2B on 12/06/2018 23:27:15
My understanding is that it is virtual particles that pop in and out of existence; and that Matt Strassler advises not thinking of them as particles.
Yes, even Feynman says not to think of them as real. General consensus is that they are seriously misnamed
 
The vacuum is a mass of virtual particles constantly popping in and out of existence.  These are not different particles, constantly being created and destroyed; rather they exist as the vacuum energy that is always present.  If this interpretation were correct, every particle would exist, in some form, at all times. It would never be “nothing”, even for an “infinitesimally short time”.
It would be hard to say they were ‘nothing’. Experiments suggest that to make them persist requires a great deal of energy. Energy is not nothing, it has to be in the form of something.

There is no doubt that what existed before this universe began was different.

You might like to read sec 6 p14 of this https://arxiv.org/pdf/hep-th/0702178.pdf
Someone was asking about Alan Guth & inflation, can’t remember who.
Title: Re: What is Nothingness?
Post by: guest45734 on 13/06/2018 10:20:36
There is no doubt that what existed before this universe began was different.

Our universe exists in an expanding space, to answer what is nothing, you have to define what space is. Space time does not exist without quantum fluctuations, some theories argue that the expansion of space is driven by quantum foam. So in defining nothingness, what would space time be without quantum foam.
It is possible (???) that gravity is caused by the absorption of space/quantum foam also. Following that reasoning inside a BH quantum foam may be absorbed completely creating a void in space time. Before the initial BB event a void in space may have existed that made the BB event/expansion of space inevitable. 

To define nothing you have to define exactly what space is, and what dimensions it has. Then you need to define just how stable it is, and if it is unstable how inevitable was the BB/expansion space from this nothing.
Title: Re: What is Nothingness?
Post by: Bogie_smiles on 13/06/2018 12:27:04
Reply #7


There is no doubt that what existed before this universe began was different.

Our universe exists in an expanding space, to answer what is nothing, you have to define what space is. Space time does not exist without quantum fluctuations, some theories argue that the expansion of space is driven by quantum foam. So in defining nothingness, what would space time be without quantum foam.
It is possible ( ??? ) that gravity is caused by the absorption of space/quantum foam also. Following that reasoning inside a BH quantum foam may be absorbed completely creating a void in space time. Before the initial BB event a void in space may have existed that made the BB event/expansion of space inevitable. 

To define nothing you have to define exactly what space is, and what dimensions it has. Then you need to define just how stable it is, and if it is unstable how inevitable was the BB/expansion space from this nothing.
Some of your post is getting off topic in regard to speculations, and might be too close to ideas that are generally better addressed in New Theories.

The definition of "nothingness" was offered in the OP, and I asked if there were changes to that definition to improve it. Do you want to change or modify the stated definition of nothingness?
Title: Re: What is Nothingness?
Post by: Bogie_smiles on 13/06/2018 12:39:45
Reply #8


You might like to read sec 6 p14 of this https://arxiv.org/pdf/hep-th/0702178.pdf (https://arxiv.org/pdf/hep-th/0702178.pdf)
[Eternal Inflation and its Implications]

Someone was asking about Alan Guth & inflation, can’t remember who.

I mentioned it in a thread in New Theories, by TheBox, “Wave Particle Duality”, and you commented re. Alan Guth.

I was researching Eternal Inflation a few years ago, and just refreshed my memory on the following:
Two papers by Laura Mersini-Houghtion; both of them reference Guth’s “Eternal Inflation and its Implications”

These papers contradict the idea that Inflation has an eternal past, i.e. it takes the position that Eternal Inflation might not be eternal as suggest in section 7 of A. H. Guth’s paper, “Eternal Inflation and its Implications”. Both Mersini-Houghton papers make multiple references to A. H. Guth’s papers on Eternal Inflation.

http://arxiv.org/pdf/1106.3542v1.pdf (http://arxiv.org/pdf/1106.3542v1.pdf)
Is Eternal Inflation Eternal?
Laura Mersini-Houghton
Is Eternal Inflation Eternal ?
L. Mersini-Houghton Department of Physics and Astrononmy, UNC-Chapel Hill, NC, 27599-3255, USA and,
CITA, University of Toronto, Canada (Dated: June 5, 2018)
Abstract:
In this paper we explore the relationship between the existence of eternal inflation and the initial conditions leading to inflation. We demonstrate that past and future completion of inflation is related, in that past-incomplete inflation can not be future eternal. Bubble universes nucleating close to the initial conditions hypersurface have the largest Lorentz boosts and experience the highest anisotropy. Consequently, their probability to collide upon formation is one. Thus instead of continuing eternally inflation ends soon after it starts. The difficulty in actualizing eternal inflation originates from the breaking of two underlying symmetries: Lorentz invariance and the general covariance of the theory which lead to an inconsistency of Einstein equations. Eternal inflation may not be eternal.


http://arxiv.org/pdf/1211.1347v1.pdf (http://arxiv.org/pdf/1211.1347v1.pdf)
End of Eternal Inflation
The End of Eternal Inflation
Laura Mersini-Houghton and Malcolm J Perry
DAMTP, University of Cambridge, Wilberforce Rd., Cambridge, CB3 0WA, England and Department of Physics and Astronomy, UNC Chapel Hill, NC 27599, USA. (Dated: November 7, 2012)
Abstract:
We propose a new measure for eternal inflation that includes both conditions, large field fluctuations and smooth homogeneous domains, in the self reproducing probability estimate. We show that due to the increasing inhomogeneities in the background spacetime fractal, self-reproductions stops within a finite time tf , thus inflation can not be eternal.


These papers contradict the idea the Inflation has an eternal past, i.e. it takes the position that Eternal Inflation might not be eternal (A. H. Guth Inflationary Theory). Both papers make multiple references to A. H. Guth’s papers on Eternal Inflation.

Title: Re: What is Nothingness?
Post by: Bogie_smiles on 14/06/2018 18:48:56
Reply #9


These papers contradict the idea the Inflation has an eternal past, i.e. it takes the position that Eternal Inflation might not be eternal (A. H. Guth Inflationary Theory). Both papers make multiple references to A. H. Guth’s papers on Eternal Inflation.
Yes, this is Alan Guth’s early paper which I thought @Bill S  might like to see, it gives an idea of the types of argument being put forward.
Since then there have been a number of papers some supporting others offering alternatives. Big work in progress, lots of very detailed analysis, not a top level subject.

If you look at the Alan Guth paper you will see just how unnothing the vacuum is. Is nothing sacred?  :)
Lol.
Lots of on going analysis, new tools, instruments, scientific advances, and views/models of the cosmology of the universe … and some express the view that we may never know for sure, but one thing is likely, and that is that the consensus will evolve as advances are made.

Still, my intent here is to air out a view of what nothingness really means, and give anyone who is interested a chance to modify it.

If nothingness is well described by the definition in the OP, then any cosmology that says nothingness preceded our observable universe will be hard pressed to explain how something can come from nothingness.



Title: Re: What is Nothingness?
Post by: Colin2B on 14/06/2018 23:11:12
One definition of nothingness: No space, no time, no energy, and no potential for any space, time or energy.
I think by including “and no potential for any space, time or energy.“ you have pre-defined nothing as unable to create anything. If a room contains no furniture I can bring some in through the door, but to say it has no potential for furniture might suggest it has no door. 
Most would say “No space, no time, no energy”
Title: Re: What is Nothingness?
Post by: Bogie_smiles on 15/06/2018 01:01:33
Reply #11


I think by including “and no potential for any space, time or energy” you have pre-defined nothing as unable to create anything. If a room contains no furniture I can bring some in through the door, but to say it has no potential for furniture might suggest it has no door. 
Most would say “No space, no time, no energy”
That is true. “No space, no time, no energy” leaves a loophole that I am trying to close.

Why close it? If there is no space, time, or energy, but there is a potential for space, time, or energy, then that opens a door to “bring in the furniture”; you didn’t have nothingness to begin with, you had the potential for something.

Instead of having to explain how you got furniture from nothingness, all you have to do is say it came from another dimension; maybe you can posit an infinite number of dimensions.

Or maybe you can use the loophole to explain that nothingness is really two opposites that can spontaneously separate. Maybe the nothingness can split, annihilate itself, and split again; you might get a cyclical action going .

Opportunity for fanciful speculations abound, and brilliant mathematicians can show how it might be possible if there is any door at all.

Title: Re: What is Nothingness?
Post by: Colin2B on 15/06/2018 07:35:56
Instead of having to explain how you got furniture from nothingness, all you have to do is say it came from another dimension; maybe you can posit an infinite number of dimensions.
But doesn’t that imply a door, which means a potential for ....

Or maybe you can use the loophole to explain that nothingness is really two opposites that can spontaneously separate. Maybe the nothingness can split, annihilate itself, and split again; you might get a cyclical action going .[/font][/size]
Two opposites are still something.

However, you raise an interesting question. Is it that we just can’t see the furniture at times!
Title: Re: What is Nothingness?
Post by: Bogie_smiles on 15/06/2018 11:08:20
Reply #13



But doesn’t that imply a door, which means a potential for ....
Yes, the door is there unless you don't open it to begin with, and my definition avoids any such door. There are no doors left if at first you have nothingness as defined in the OP.

Quote
Two opposites are still something.
Exactly. That is why I include "and no potential for space, time or energy" in the definition of nothingness. The intention is to leave no room for anything to come from nothingness.
Quote
However, you raise an interesting question. Is it that we just can’t see the furniture at times!
That is not my point though. I offered those scenarios to show what kind of speculations can be posited if there is any hint of a door in the definition of nothingness; I wasn't suggesting them as viable options. If the definition includes the language of " and no potential", then those cases I mentioned would not stand up simply because they violate the definition of nothingness.
Title: Re: What is Nothingness?
Post by: Colin2B on 15/06/2018 14:03:03
But that’s probably not everyone’s definition of nothing. Important to check that definition in each theory.
Title: Re: What is Nothingness?
Post by: Bogie_smiles on 15/06/2018 14:52:31
Reply #15


But that’s probably not everyone’s definition of nothing. Important to check that definition in each theory.
That would work for me, if you mean that any scientific model of cosmology should include a paragraph that deals with the issue of the beginning that the model invokes. A case in point, using the current status of what the scientific community might agree was the BBT model, i.e. GR with Inflation, a paragraph could be proposed and tested among those in the scientific community with the intent of finding a consensus on "how the model addresses the issue of the beginning".


Maybe it would say that the consensus is that we really don't know much about before ~10^-43 seconds, except that we can portray that instant as a hot, dense wave of energy in the form of a plasma emerging from a preceding condition, i.e.,the consensus is that there were preconditions before the time at 10^-43 seconds, and we are not invoking the "something from nothing" explanation for the existence of the universe at that point in time. (Or some such wording, :) ). Do you think that would fly?
Title: Re: What is Nothingness?
Post by: Bogie_smiles on 15/06/2018 15:39:40
Reply #16



The first part might not present too great a problem; but I genuinely wish you luck with the second part.
Point well taken.
Quote
I'll be quite happy to chip in later, but I think there is a tendency to run away when I climb on my soapbox about "something from nothing", so I'll hold off rather than put your chances at risk.
Lol, hopefully by now you see where I am coming from. Whether or not we have some common ground on this issue, your view on the applicability of my definition of nothingness would be valued.
Title: Re: What is Nothingness?
Post by: Colin2B on 15/06/2018 16:29:43
@Bogie_smiles , “Do you think that would fly?”
It would certainly help. It is very clear in Alan Guth’s papers as he states that he is not dealing with what happens before inflation.

Are you volunteering to produce a chart showing what assumptions are made in various models.  :)
Title: Re: What is Nothingness?
Post by: Bogie_smiles on 15/06/2018 18:02:33
Reply #18



It would certainly help. It is very clear in Alan Guth’s papers as he states that he is not dealing with what happens before inflation.

Are you volunteering to produce a chart showing what assumptions are made in various models.  :)
I wish I was that smart.

In support of a "beginnings disclaimer" paragraph for each model though, it would be for the benefit of professional peers and layman enthusiasts, but ideally those professionals submitting the model should address their stance on preconditions, the beginning, or acknowledging "we just don't know" much about "before" where the model picks up.

It would provide a partition between what the model is presenting, and what all of the peers and layman might speculate that the preconditions to the model might be.
Title: Re: What is Nothingness?
Post by: Bogie_smiles on 17/06/2018 17:57:16
Reply #19



My understanding is that it is virtual particles that pop in and out of existence; and that Matt Strassler advises not thinking of them as particles.

Be that as it may, these virtual particles borrow energy from the vacuum for such a short time that it the vacuum “doesn’t notice”.  Putting it that way leaves me with the question: if the “particle” was nothing before it popped into existence, what borrowed the energy?
Agreed, there was something to cause the event of a virtual particle “popping” into observability. That something, in accord with the definition of nothingness in the OP, was a potential energy, and it has the potential to form a virtual particle under certain conditions that are in accord with the invariant laws of nature.

Title: Re: What is Nothingness?
Post by: Bill S on 17/06/2018 22:53:20
Quote from: Bogie smiles
a paragraph could be proposed and tested among those in the scientific community with the intent of finding a consensus on "how the model addresses the issue of the beginning".

Sadly, what we often seem to need is an explanation as to whether the "author" thinks "nothing" is "nothing" or "something" 
Title: Re: What is Nothingness?
Post by: PmbPhy on 17/06/2018 22:56:22
What is nothingness?
The cause of my stomach making funny sounds??
Title: Re: What is Nothingness?
Post by: Bill S on 17/06/2018 23:27:31
Nice one Pete. Promise I'll credit you if I ever use it.  :)
Title: Re: What is Nothingness?
Post by: Bogie_smiles on 17/06/2018 23:39:15

The cause of my stomach making funny sounds??


Are you suggesting that I modify the definition of nothingness in the OP, perhaps to:
No space, no time, no energy, and no hunger sounds,  and no potential for space, time, or energy

I don't see it as an improvement.

Title: Re: What is Nothingness?
Post by: Bogie_smiles on 17/06/2018 23:40:14
Reply #24



Sadly, what we often seem to need is an explanation as to whether the "author" thinks "nothing" is "nothing" or "something" 
That would be nice to know.
Title: Re: What is Nothingness?
Post by: Colin2B on 18/06/2018 13:28:59
@PmbPhy makes a good point
There is the same amount of energy before the big bang as there is now. The total amount of energy in the universe is zero. That's how energy remain conserved as it was created. There's two kinds of energy; positive and negative. Negative energy comes from gravitational potential energy and positive energy comes from particles. You can read about it in The Inflationary Universe by Alan Guth. I can make those pages available to you if you'd like?
So even if the energy pre-whatever is zero, it doesn’t imply nothing.
Title: Re: What is Nothingness?
Post by: PmbPhy on 18/06/2018 13:29:48
Quote from: Bogie_smiles
Are you suggesting that I modify the definition of nothingness in the OP, perhaps to:
No space, no time, no energy, and no hunger sounds,  and no potential for space, time, or energy
If that's what you do with responses meant to give people a reason to smile then sure, why not.
Title: Re: What is Nothingness?
Post by: PmbPhy on 18/06/2018 13:31:33
Quote from: Colin2B
So even if the energy pre-whatever is zero, it doesn’t imply nothing.
I don't know. I have no physical example to base a reliable response on.
Title: Re: What is Nothingness?
Post by: Bogie_smiles on 18/06/2018 13:38:18
Reply #28




Sadly, what we often seem to need is an explanation as to whether the "author" thinks "nothing" is "nothing" or "something" 

Agreed, and in fact what modern cosmology is doing is exploring what isn’t nothing, i.e., what is “Somethingness”; what are the alternatives to Nothingness as defined in the OP.

As pointed by Colin2B, the alternatives don’t generally state that they are intentionally invoking the “Something from nothing” explanation, and so they are likely to ignore the issue. Taking a practical look at the development of modern cosmology, we could start with the definition of the Cosmological Principle:

https://en.wikipedia.org/wiki/Cosmological_principle (https://en.wikipedia.org/wiki/Cosmological_principle)
In modern physical cosmology, the cosmological principle is the notion that the spatial distribution of matter in the universe is homogeneous and isotropic when viewed on a large enough scale, since the forces are expected to act uniformly throughout the universe, and should, therefore, produce no observable irregularities in the large-scale structuring over the course of evolution of the matter field that was initially laid down by the Big Bang.

That explanation points to a Cosmological Constant or Omega value of 1. (https://en.wikipedia.org/wiki/Cosmological_constant)

The Friemann equations address the expansion of space within the context of Einstein’s General Relativity:
https://en.wikipedia.org/wiki/Friedmann_equations (https://en.wikipedia.org/wiki/Friedmann_equations)
The Friedmann equations are a set of equations in physical cosmology that govern the expansion of space in homogeneous and isotropic models of the universe within the context of general relativity. They were first derived by Alexander Friedmann in 1922[1] from Einstein's field equations of gravitation for the Friedmann–Lemaître–Robertson–Walker metric and a perfect fluid with a given mass density

Note the reference to a perfect fluid with a given mass density:
https://en.wikipedia.org/wiki/Perfect_fluid (https://en.wikipedia.org/wiki/Perfect_fluid)
In physics, a perfect fluid is a fluid that can be completely characterized by its rest frame mass density
ρ, m; and
isotropicpressurep.
Real fluids are "sticky" and contain (and conduct) heat. Perfect fluids are idealized models in which these possibilities are neglected. Specifically, perfect fluids have no shear stresses, viscosity, or heat conduction.


Alan Guth, mentioned above in Colin2B’s link, didn’t see the observational results of a perfect fluid and set out to understand the formation of large scale structure in the universe.That is why the field of modern cosmology leads to Alan Guth
https://en.wikipedia.org/wiki/Alan_Guth (https://en.wikipedia.org/wiki/Alan_Guth)
See the section on Inflationary theory.
Title: Re: What is Nothingness?
Post by: Colin2B on 18/06/2018 13:43:37
Quote from: Colin2B
So even if the energy pre-whatever is zero, it doesn’t imply nothing.
I don't know. I have no physical example to base a reliable response on.
I think that’s the conclusion this thread is coming to. There are many unknowns and no reliable examples, but I think @Bogie_smiles is just trying to clarify that many of the interpretations of something from nothing are not based on clear definitions of what is truly nothing - or isn’t?  :)

PS hunger sounds implies a hungry tum, that’s probably because there’s ‘nothing’ in it ;)
Title: Re: What is Nothingness?
Post by: Bogie_smiles on 18/06/2018 17:58:11
Reply #30


So even if the energy pre-whatever is zero, it doesn’t imply nothing.

I think that’s the conclusion this thread is coming to. There are many unknowns and no reliable examples, but I think @Bogie_smiles is just trying to clarify that many of the interpretations of something from nothing are not based on clear definitions of what is truly nothing - or isn’t?  :)


Correct. That is why I include the phrase, “and no potential for space, time, or energy” in the definition.

If there was nothingness, and no potential for anything, then you can refer to that as zero energy if you want because there is zero energy, but you cannot call it a pre-energy state; a pre-zero energy state is somethingness :) and cannot come from nothingness.

Title: Re: What is Nothingness?
Post by: Bogie_smiles on 18/06/2018 18:18:10
Reply #31



In reply #28, re. taking a practical look at the development of modern cosmology, it doesn’t take long to get to General Relativity and Inflationary theory.

Here is a cut and paste of the section in the Guth Wiki that addresses Inflationary theory, if you want to take a quick read:

Guth’s first step to developing his theory of inflation occurred at Cornell in 1978, when he attended a lecture by Robert Dicke about the flatness problem of the universe.[7] Dicke explained how the flatness problem showed that something significant was missing from the Big Bang theory at the time. The fate of the universe depended on its density. If the density of the universe was large enough, it would collapse into a singularity, and if the actual density of the matter in the cosmos was lower than the critical density, the universe would increasingly get much bigger.
The next part in Guth's path came when he heard a lecture by Steven Weinberg in early 1979.[8] Weinberg talked in two lectures about the Grand Unified Theory (GUT) that had been developed since 1974, and how it could explain the huge amount of matter in the universe compared to the amount of antimatter. The GUT explained all the fundamental forces known in science except for gravity. It established that in very hot conditions, such as those after the Big Bang, electromagnetism, the strong nuclear force, and the weak nuclear force were united to form one force. Weinberg also was the one who emphasized the idea that the universe goes through phase transitions, similar to the phases of matter, when going from high energy to low energy. Weinberg’s discussion of why matter is so dominant over anti-matter showed Guth how precise calculations about particles could be obtained by studying the first few seconds of the universe.
Guth decided to solve this problem by suggesting a supercooling during a delayed phase transition. This seemed very promising for solving the magnetic monopole problem. By the time they came up with that, Guth had gone to the Stanford Linear Accelerator Center (SLAC) for a year, but Guth had been talking to Henry Tye back and forth. Tye suggested that they check that the expansion of the universe would not be affected by the supercooling. In the supercooled state, a false vacuum is produced. The false vacuum is a vacuum in the sense that it is the state of the lowest possible density of energy; it is false in the sense that it is not a permanent state of being. False vacuums decay, and Guth was to find that the decay of the false vacuum at the beginning of the universe would produce amazing results, namely the exponential expansion of space. This solved the monopole problem, since the expansion dilutes the monopole density.
Guth realized from his theory that the reason the universe appears to be flat was that it was fantastically big, just the same way the spherical Earth appears flat to those on its surface. The observable universe was actually only a very small part of the actual universe. Traditional Big Bang theory found values of omega near one to be puzzling, because any deviations from one would quickly become much, much larger. In inflation theory, no matter where omega starts, it would be driven towards equal to one, because the universe becomes so huge. In fact, a major prediction of inflationary theory is that omega will be found to be one.
Two weeks later, Guth heard colleagues discussing something called the horizon problem. The microwave background radiation discovered by Arno Penzias and Robert Woodrow Wilson appeared extremely uniform, with almost no variance. This seemed very paradoxical because, when the radiation was released about 300,000 years after the Big Bang, the observable universe had a diameter of 90 million light-years. There was no time for one end of the cosmos to communicate with the other end, because energy can not move faster than the speed of light. The paradox was resolved, as Guth soon realized, by the inflation theory. Since inflation started with a far smaller amount of matter than the Big Bang had presupposed, an amount so small that all parts would have been in touch with each other. The universe then inflated at billion times the speed of light so the homogeneity remained unbroken. The universe after inflation would have been very uniform even though the parts were not still in touch with each other.
Guth first made public his ideas on inflation in a seminar at SLAC in January 1980. He ignored magnetic monopoles because they were based on assumptions of GUT, which was outside the scope of the speech. In August, he submitted his paper, entitled "Inflationary universe: A possible solution to the horizon and flatness problems" to the journal Physical Review.[9] In this paper Guth postulated that the inflation of the universe could be explained if the universe were supercooled 28 orders of magnitude below the critical temperatures required for a phase change.
In December 1981, Guth read a paper from Moscow physicist Andrei Linde saying that the whole universe is within just one bubble, so nothing is destroyed by wall collisions. This conclusion was made using a Higgs field with an energy graph that was originally proposed by Sidney Coleman and Erick Weinberg. Guth discussed this with Linde, who had independently been working on bubble inflation, but without considering the flatness problem. Linde and Guth eventually exchanged papers on the subject.
By 1983 Guth had published a paper describing how his supercooled-universe scenario was not ideal, as the "triggering mechanism" to exit such a state would require "extreme fine tuning of parameters" and felt a more natural solution was required.[1][10][11] However, this did not deter him from the belief that the universe expanded exponentially in a vacuum in its early lifetime.[12]
[End of Wiki cut and paste]

Note "Henry Tye suggested that they check that the expansion of the universe would not be affected by the supercooling. In the supercooled state, a false vacuum is produced. The false vacuum is a vacuum in the sense that it is the state of the lowest possible density of energy; it is false in the sense that it is not a permanent state of being. False vacuums decay, and Guth was to find that the decay of the false vacuum at the beginning of the universe would produce amazing results, namely the exponential expansion of space".

That is not a perfect vacuum of course, and it is not nothingness.

To be continued ...
Title: Re: What is Nothingness?
Post by: Bogie_smiles on 19/06/2018 11:56:49
Reply #32

The Wiki information is saying that the findings of Alan Guth were that the decay of the false vacuum at the beginning of the universe would produce exponential expansion of space. Have you given Guth’s papers a good read to understand the particulars of Inflationary theory? I’m wondering about the mechanics of the decay of the false vacuum and the resulting exponential expansion of space.


@Bogie_smiles , “Do you think that would fly?”
It would certainly help. It is very clear in Alan Guth’s papers as he states that he is not dealing with what happens before inflation.

...
Does he say what causes a false vacuum in the first place? Does he say there was a hot dense plasma at the beginning of the expansion? Where does that come from?
Title: Re: What is Nothingness?
Post by: Bogie_smiles on 23/06/2018 22:55:37
Reply #33


Here are a few links that I have searched that begin to shed some light on the false vacuum, and the theories where it applies.

I find it helps my thought process if I cut and paste content from the links. See my discussion below the linked content:
vvvvvvvvvvvvvvvvvvvvvvvv
https://en.m.wikipedia.org/wiki/False_vacuum (https://en.m.wikipedia.org/wiki/False_vacuum)

In quantum field theory, a false vacuum is a hypothetical vacuum that is somewhat, but not entirely, stable. It may last for a very long time in that state, and might eventually move to a more stable state. The most common suggestion of how such a change might happen is called bubble nucleation - if a small region of the universe by chance reached a more stable vacuum, this 'bubble' would spread.
A false vacuum may only exist at a local minimum of energy and is therefore not stable, in contrast to a true vacuum, which exists at a global minimum and is stable. A false vacuum may be very long-lived, or metastable.


bubble nucleation

In the theoretical physics of the false vacuum, the system moves to a lower energy state – either the true vacuum, or another, lower energy vacuum – through a process known as bubble nucleation.[4][5][24][25][26][27] In this, instanton (https://en.m.wikipedia.org/wiki/Instanton (https://en.m.wikipedia.org/wiki/Instanton)) effects cause a bubble to appear in which fields have their true vacuum values inside. Therefore, the interior of the bubble has a lower energy. The walls of the bubble (or domain walls) have a surface tension, as energy is expended as the fields roll over the potential barrier to the lower energy vacuum. The most likely size of the bubble is determined in the semi-classical approximation to be such that the bubble has zero total change in the energy: the decrease in energy by the true vacuum in the interior is compensated by the tension of the walls.
Joseph Lykken has said that study of the exact properties of the Higgs boson could shed light on the possibility of vacuum collapse.[28]
__________

Instanton: An instanton[1] (or pseudoparticle[2][3]) is a notion appearing in theoretical and mathematical physics. An instanton is a classical solution to equations of motion[note 1] with a finite, non-zero action, either in quantum mechanics or in quantum field theory. More precisely, it is a solution to the equations of motion of the classical field theory on a Euclidean spacetime.
In such quantum theories, solutions to the equations of motion may be thought of as critical points of the action. The critical points of the action may be local maxima of the action, local minima, or saddle points. Instantons are important in quantum field theory because:
they appear in the path integral as the leading quantum corrections to the classical behavior of a system, and
they can be used to study the tunneling behavior in various systems such as a Yang–Mills theory.
In dynamics, instantons are families of deterministic solutions that connect, e.g., different critical points of equations of motion. From the physical point of view, instantons are particularly important because the condensation of instantons (and noise-induced anti-instantons) is believed to be the theoretical essence of the noise-induced chaotic phase known also as self-organized criticality.




Any increase in size of the bubble will decrease its potential energy, as the energy of the wall increases as the surface area of a sphere
4
π
r
2
{\displaystyle 4\pi r^{2}}
<img src="https://wikimedia.org/api/rest_v1/media/math/render/svg/b81fcce302776a01dc66fc186a1ce0a616b4d772 (https://wikimedia.org/api/rest_v1/media/math/render/svg/b81fcce302776a01dc66fc186a1ce0a616b4d772)" class="mwe-math-fallback-image-inline" aria-hidden="true" style="vertical-align: -0.338ex; width:4.597ex; height:2.676ex;" alt="4\pi r^{2}"> but the negative contribution of the interior increases more quickly, as the volume of a sphere
4
3
π
r
3
{\displaystyle \textstyle {\frac {4}{3}}\pi r^{3}}
<img src="https://wikimedia.org/api/rest_v1/media/math/render/svg/d685ddd147c84e123fe4c01d6a7ee9813f7b04d3 (https://wikimedia.org/api/rest_v1/media/math/render/svg/d685ddd147c84e123fe4c01d6a7ee9813f7b04d3)" class="mwe-math-fallback-image-inline" aria-hidden="true" style="vertical-align: -1.338ex; width:5.093ex; height:3.676ex;" alt="\textstyle {\frac {4}{3}}\pi r^{3}">. Therefore, after the bubble is nucleated, it quickly begins expanding at very nearly the speed of light. The excess energy contributes to the very large kinetic energy of the walls. If two bubbles are nucleated and they eventually collide, it is thought that particle production would occur where the walls collide.
The tunnelling rate is increased by increasing the energy difference between the two vacua and decreased by increasing the height or width of the barrier.


The addition of gravity to the story leads to a considerably richer variety of phenomena. The key insight is that a false vacuum with positive potential energy density is a de Sitter vacuum, in which the potential energy acts as a cosmological constant and the Universe is undergoing the exponential expansion of de Sitter space. This leads to a number of interesting effects, first studied by Coleman and de Luccia.[3]


https://en.m.wikipedia.org/wiki/Maxima_and_minima (https://en.m.wikipedia.org/wiki/Maxima_and_minima)

In mathematical analysis, the maxima and minima (the respective plurals of maximum and minimum) of a function, known collectively as extrema (the plural of extremum), are the largest and smallest value of the function, either within a given range (the local or relative extrema) or on the entire domain of a function (the global or absolute extrema).[1][2][3] Pierre de Fermat was one of the first mathematicians to propose a general technique, adequality, for finding the maxima and minima of functions.




If the Standard Model is correct, the particles and forces we observe in our universe exist as they do because of underlying quantum fields. Quantum fields can have states of differing stability, including 'stable', 'unstable', or 'metastable' (meaning very long-lived but not completely stable). If a more stable vacuum state were able to arise, then existing particles and forces would no longer arise as they do in the universe's present state. Different particles or forces would arise from (and be shaped by) whatever new quantum states arose. The world we know depends upon these particles and forces, so if this happened, everything around us, from subatomic particles to galaxies, and all fundamental forces, would be reconstituted into new fundamental particles and forces and structures. The universe would lose all of its present structures and become inhabited by new ones (depending upon the exact states involved) based upon the same quantum fields.



^^^^^^^^^^^^^^^^^^^^^^

Discussion; The false vacuum is not entirely stable, but by chance it may move to a more stable state:

“The most common suggestion of how such a change [to a more stable state] might happen is called bubble nucleation - if a small region of the universe by chance reached a more stable vacuum, this 'bubble' would spread.”

Note that the false vacuum is local minimum energy, and theoretically, can move to a more stable state, i.e., contain more potential energy  per volume of space than the false vacuum, by bubble nucleation. It becomes a smaller bubble within the false vacuum bubble and therefore has higher energy potential.

Bubbles can expand, and if they do, their potential energy decreases down toward the energy potential of the local false vacuum.

If two bubbles are nucleated within a false vacuum, and they eventually collide, it is thought that particle production would occur where the walls collide. Is that a potential source of the hot dense plasma that is consistent with Standard theory at 10^-43?


It would seem to be consistent with QFT, at least per the limited content of the above links.


Title: Re: What is Nothingness?
Post by: Bill S on 23/06/2018 23:28:25
It never ceases to amaze me, how complex a discussion about "nothing" can become. 
Isn't a false vacuum "something".

I'm looking for a bit by Vilenkin on that subject, but I'll not have time to find it tonight.
Title: Re: What is Nothingness?
Post by: Bogie_smiles on 23/06/2018 23:46:43
Reply #35

Perhaps you are seeing my point, that there is no chance of nothingness, since there is obviously something; many fine things all around us for that matter. None of which would be possible if at first there was nothing, at least under the definition I gave in the OP.
It never ceases to amaze me, how complex a discussion about "nothing" can become. 
Isn't a false vacuum “something".
Yes indeed. Though it is only theoretical, as you can see, following two theories, BBT with Inflation, and QFT at the barest level of investigation, both are consistent with the formation of matter out of “bubble convergences”.

Title: Re: What is Nothingness?
Post by: Bill S on 24/06/2018 13:14:35
Quote
Perhaps you are seeing my point, that there is no chance of nothingness

And you mine (?)

Quote
There never was nothing.

What do they say about great minds? :)

Quote
BBT with Inflation, and QFT at the barest level of investigation, both are consistent with the formation of matter out of “bubble convergences”.

That's fine, as long as you interpret "bubbles" as "something".

Title: Re: What is Nothingness?
Post by: Bill S on 24/06/2018 14:30:38
I’ve found the Vilenkin reference, but before looking at it I would like an opinion as to the accuracy, at a non-technical level, of my understanding of the idea of representing the vacuum as a “landscape”.

In trying to establish a non-technical image of the vacuum, and its energy, it may be helpful to visualise it as an irregular surface of waves and troughs which represent this energy, and to consider this as a “landscape”. 

The bottom of the lowest valley would be considered to be the “true vacuum”, that in which our Universe exists.  Crests of waves, and the slopes linking them to the valleys, represent “false “vacuum” states in which the vacuum energy is higher than that of the true vacuum.  Essentially, these are very unstable states.  Some stability may be achieved by a false vacuum state, and this would be represented on the vacuum landscape by a valley at a higher level than the true vacuum. 

Obviously, this gives some stability to the false vacuum state, and to see why, it is necessary to look at the “inhabitants” of the vacuum landscape.  It appears that these inhabitants are scalar fields.  Popular science books usually depict these scalar fields as spheres that can roll about on the hilly landscape.  Their energy is dictated by the vertical position they occupy, at any given instant, on the landscape.  A scalar field that is at, or near, the top of a hill will equate to a false vacuum state.  It will be unstable, so will have a tendency to “roll down” to a lower level if an opportunity presents itself.  Energy would be needed to "lift" the scalar field out of the valley so that it could progress to a lower level.

Title: Re: What is Nothingness?
Post by: Bogie_smiles on 24/06/2018 17:36:43
Reply #38




And you mine (?)


What do they say about great minds? :)
They are something, right :)
Quote
That's fine, as long as you interpret "bubbles" as "something".
Two bubbles nucleated within a false vacuum in QFT are something, as is Guth's supercooled vacuum at the start of Inflationary theory, and certainly BBT's hot dense ball of energy, all are something.

I’ve found the Vilenkin reference, but before looking at it I would like an opinion as to the accuracy, at a non-technical level, of my understanding of the idea of representing the vacuum as a “landscape”.


In trying to establish a non-technical image of the vacuum, and its energy, it may be helpful to visualise it as an irregular surface of waves and troughs which represent this energy, and to consider this as a “landscape”. 


The bottom of the lowest valley would be considered to be the “true vacuum”, that in which our Universe exists.  Crests of waves, and the slopes linking them to the valleys, represent “false “vacuum” states in which the vacuum energy is higher than that of the true vacuum.  Essentially, these are very unstable states.  Some stability may be achieved by a false vacuum state, and this would be represented on the vacuum landscape by a valley at a higher level than the true vacuum. 


Obviously, this gives some stability to the false vacuum state, and to see why, it is necessary to look at the “inhabitants” of the vacuum landscape.  It appears that these inhabitants are scalar fields.  Popular science books usually depict these scalar fields as spheres that can roll about on the hilly landscape.  Their energy is dictated by the vertical position they occupy, at any given instant, on the landscape.  A scalar field that is at, or near, the top of a hill will equate to a false vacuum state.  It will be unstable, so will have a tendency to “roll down” to a lower level if an opportunity presents itself.  Energy would be needed to "lift" the scalar field out of the valley so that it could progress to a lower level.
That would be something, with some emphasis.


QFT seems to be a work in progress, and like string theory, got traction from the shortcomings of BBT, especially the infinitely dense, zero volume conclusion reached by backtracking the observed expansion to its limit.


If I could weigh in on your non technical understanding of the landscape of the greater universe, in regard to visualizing it from the QFT perspective, it would seem permissible to discuss the nucleating bubbles across the true vacuum as a landscape. Is this an infinite landscape that would fill all space? If not, I suspect nothingness would want to pop up in the "what lies beyond".


Let's see the Vilenkin link.
Title: Re: What is Nothingness?
Post by: Bill S on 24/06/2018 23:42:33
Quote
Let's see the Vilenkin link.

It's not a link, I'm afraid; just some notes from a good-old-fashioned book.

With any luck I might have a half-hour to sort the notes out now.
Title: Re: What is Nothingness?
Post by: Bill S on 25/06/2018 18:28:29
No luck with the time last night, but I've cobbled something together.

On the subject of the scalar fields; one question must be: how big are they? 

This is a question that is commonly ignored, possibly with good reason, but in developing his argument, Vilenkin regards them as quantum objects, but he also talks of “a false vacuum at every point in space”, so it is probably reasonable to assume that he does not equate these scalar fields with the false vacuum.  That would seem to make good sense, but is not always apparent in popular science writing.

    Imagine a landscape without a false vacuum “valley”.  There is a sphere at, or near the top of a very gently sloping crest.  It is a quon, Vilenkin says, it will be subject to quantum “kicks” which cause it to move about randomly.  Eventually it will migrate downslope, which will trigger inflation.  The bottom of the valley into which the sphere rolls is considered to be the true vacuum, and this is where the scalar field discharges its energy. 

    Next consider a false vacuum “valley”, with the sphere resting in the bottom.  Quantum theory tells us that the energy input needed to lift the sphere over the energy barrier can be eliminated if the sphere tunnels through the barrier.  Such an occurrence is vanishingly unlikely if the sphere is anything other than a quon; but a quon is what we are visualising. 

Vilenkin states: “Despite the similarity between the tunneling of a ball and that of a scalar field, there is an important difference.  The ball tunnels between two different points in space, while for the scalar field the tunneling is between two different values of the field at the same location.”  Presumably, “location” can be taken to mean “at every point in space”.

  A single quon with the immense energy that it derives from its position in false energy value, tunnels to a lower energy value, and in so doing, gives birth to a universe.  If eternal inflation is the answer, this has always been happening, and will always happen.  There has always been a vacuum with a non-zero vacuum energy, and there always will be.  Quantum scale universes have been tunnelling through energy barriers and “erupting” eternally in the past, and they will continue to do so, infinitely, into the future. 

    Vilenkin formulates a seductive argument for reducing his initial quon to nothing, thus having “nothing” tunnelling through the energy barrier and becoming something – a universe. 

    Having minimised the initial quon until there is nothing, Vilenkin says:  “The initial state prior to the tunneling is a universe of vanishing radius, that is, no universe at all.  There is no matter and no space in this very peculiar state.  Also, there is no time.  Time has meaning only if something is happening in the universe.”

    We seem to be back to a point where we are confronted with a situation in which “nothing” involves no time, so nothing can change.  There can be no tunnelling, no emergent universe and, therefore, no subsequent inflation.  Vilenkin acknowledges this, as he continues: “And yet, this state of “nothing” cannot be identified with absolute nothingness.  The tunneling is described by the laws of quantum mechanics, and thus “nothing” should be subjected to these laws.  The laws of physics must have existed, even though there was no universe.”

Vilenkin’s book (or at least this section of it) is an interesting read, and I have certainly not done it justice here.
However, he does seem to have gone to considerable lengths to demonstrate how a universe can come from nothing, only to concede that this “nothing” must be “something”.
Title: Re: What is Nothingness?
Post by: Bogie_smiles on 26/06/2018 00:13:26
Reply #41


No luck with the time last night, but I've cobbled something together.
I recognize the name now; he is the “something from nothing man”. There are quite a few links that come up in a DuckDuckGo search.

Here’s a Wiki
https://en.wikipedia.org/wiki/Alexander_Vilenkin (https://en.wikipedia.org/wiki/Alexander_Vilenkin)


Also,
https://www.sciencedirect.com/science/article/pii/0370269382908668 (https://www.sciencedirect.com/science/article/pii/0370269382908668)

Creation of universes from nothing
Author links open overlay panel
AlexanderVilenkin
Show more
https://doi.org/10.1016/0370-2693(82)90866-8 (https://doi.org/10.1016/0370-2693(82)90866-8)
Get rights and content



Abstract
A cosmological model is proposed in which the universe is created by quantum tunneling from literally nothing into a de Sitter space. After the tunneling, the model evolves along the lines of the inflationary scenario. This model does not have a big-bang singularity and does not require any initial or boundary conditions.

————————————

According to my definition, you can’t get a universe from literally nothing, so when I looked up Vilekin and saw his work, I realized I had come across his view somewhere back in time.  I am approaching your post with a skeptical view :)
Quote
1) On the subject of the scalar fields; one question must be: how big are they?

This is a question that is commonly ignored, possibly with good reason, but in developing his argument, Vilenkin regards them as quantum objects, but he also talks of “a false vacuum at every point in space”, so it is probably reasonable to assume that he does not equate these scalar fields with the false vacuum.  That would seem to make good sense, but is not always apparent in popular science writing.
I’ll go along with your thinking on that.
Quote
2)    Imagine a landscape without a false vacuum “valley”.  There is a sphere at, or near the top of a very gently sloping crest.  It is a quon, Vilenkin says, it will be subject to quantum “kicks” which cause it to move about randomly.  Eventually it will migrate downslope, which will trigger inflation.  The bottom of the valley into which the sphere rolls is considered to be the true vacuum, and this is where the scalar field discharges its energy. 
I can’t place the quon, and I don’t see it in a search “Vilenkin quon”, however, I can picture how he is using the term, from you description. The false vacuum has a randomness about it for sure. As for it being randomly kicked, and migrating downslope to trigger inflation, you know how I feel about inflation being preceded by nothing, so I’m not excited about a quon triggering inflation unless we can establish where the false vacuum came from. I expect he has an answer about cause and effect right back to the first event after nothingness, but how that first event occurred … I probably will not be open to it being something from nothing; are you?

Quote
3)    Next consider a false vacuum “valley”, with the sphere resting in the bottom.  Quantum theory tells us that the energy input needed to lift the sphere over the energy barrier can be eliminated if the sphere tunnels through the barrier.  Such an occurrence is vanishingly unlikely if the sphere is anything other than a quon; but a quon is what we are visualising. 
And if it is the quon doing it, I’m still wanting to know about the initial event in his theory after nothingness.
Quote
4) Vilenkin states: “Despite the similarity between the tunneling of a ball and that of a scalar field, there is an important difference.  The ball tunnels between two different points in space, while for the scalar field the tunneling is between two different values of the field at the same location.”  Presumably, “location” can be taken to mean “at every point in space”.
Presumably you are right about location being at every point in space. Do you know if he visualizes space to be infinite or finite?
Quote
5)  A single quon with the immense energy that it derives from its position in false energy value, tunnels to a lower energy value, and in so doing, gives birth to a universe.  If eternal inflation is the answer, this has always been happening, and will always happen.  There has always been a vacuum with a non-zero vacuum energy, and there always will be.  Quantum scale universes have been tunnelling through energy barriers and “erupting” eternally in the past, and they will continue to do so, infinitely, into the future.
Now you're talking, lol. But why is he associated with it all coming from nothingness?
Quote
6)    Vilenkin formulates a seductive argument for reducing his initial quon to nothing, thus having “nothing” tunnelling through the energy barrier and becoming something – a universe.
I don’t think I could be seduced by that reasoning, but I sense that you have some leanings. 
Quote
7)   Having minimised the initial quon until there is nothing, Vilenkin says:  “The initial state prior to the tunneling is a universe of vanishing radius, that is, no universe at all.  There is no matter and no space in this very peculiar state.  Also, there is no time.  Time has meaning only if something is happening in the universe.”
But it is not nothingness if there is a potential for all of the fine things he is  speculating about.
Quote
8)    We seem to be back to a point where we are confronted with a situation in which “nothing” involves no time, so nothing can change.  There can be no tunnelling, no emergent universe and, therefore, no subsequent inflation.  Vilenkin acknowledges this, as he continues: “And yet, this state of “nothing” cannot be identified with absolute nothingness.  The tunneling is described by the laws of quantum mechanics, and thus “nothing” should be subjected to these laws.  The laws of physics must have existed, even though there was no universe.”
That would mean that Vilekin differentiates between that state of “nothing” and absolute nothingness. If the laws of quantum mechanics describe tunneling, and if this state of “nothing” is subjected to QM laws, we are not in the same type of reality; sorry I can’t.
Quote
9) Vilenkin’s book (or at least this section of it) is an interesting read, and I have certainly not done it justice here.
However, he does seem to have gone to considerable lengths to demonstrate how a universe can come from nothing, only to concede that this “nothing” must be “something”.
Yes, that has to be the same dilemma that anyone who invokes my OP definition of nothingness will have to deal with. Maybe reading his book would help his case, but I have years of reading I would put ahead of it. Even

https://en.wikipedia.org/wiki/Ayn_Rand (https://en.wikipedia.org/wiki/Ayn_Rand) would come before that, lol.
Title: Re: What is Nothingness?
Post by: Bill S on 26/06/2018 01:05:33
Quote
I can’t place the quon, and I don’t see it in a search “Vilenkin quon”, however, I can picture how he is using the term, from you description

Actually, Vilenkin doesn't use the word "quon", but you'll find it at:

https://en.m.wikipedia.org/wiki/Quon
Title: Re: What is Nothingness?
Post by: Bill S on 26/06/2018 01:38:30
Quote
  I probably will not be open to it being something from nothing; are you?

Absolutely not.  I tend to approve of open mindedness, but not to the extent that the brain falls out. :)

Quote
I don’t think I could be seduced by that reasoning, but I sense that you have some leanings.

I can appreciate a reasoned argument, but that doesn't mean I agree with it. 

Quote
However, he does seem to have gone to considerable lengths to demonstrate how a universe can come from nothing, only to concede that this “nothing” must be “something”.

Could it be that even Vilenkin was not seduced by his own reasoning!
Title: Re: What is Nothingness?
Post by: Bogie_smiles on 26/06/2018 02:40:32
Reply #44


Absolutely not.  I tend to approve of open mindedness, but not to the extent that the brain falls out. :)

I can appreciate a reasoned argument, but that doesn't mean I agree with it. 

Could it be that even Vilenkin was not seduced by his own reasoning!
Hard to tell. Maybe so.


You have done a good job of looking at Vilenkin’s model and how he deals with the beginning, something from nothingness in his case, or how he avoids dealing with it, lol. That reminds me that Colin2B didn’t seem appalled by the idea of looking at the various models, and writing a paragraph about how they deal with the issue of a beginning, or should I say, how they avoid dealing with it. You have done that for Vilenkin’s model.

Did your analysis come from his book,  “Birth of Inflationary Universes”, or what was the title of the book you were referring to?

Perhaps one clue that he would fail to satisfy us in regard to the OP definition of nothingness is right there in the title of his book where he used the word “universes” instead of universe. To me, there is a big distinction as to whether there is just one universe, or if it is possible there could be multiple universes, and by multiple, I mean that the universes are completely separate from each other and they can never interact or influence one another. And that separation means that not even a little gravity can leak from one to another, lol.

I’d like to know how you, and or the general membership feel about this definition of universe ...

Universe: There is just one universe and it encompasses all there is, all matter, energy, everything, in one infinite and eternal presence, that had no beginning and will have no end, i.e., the universe has always existed, and has always been governed by the same set of invariant natural laws.   


Title: Re: What is Nothingness?
Post by: Bill S on 26/06/2018 19:43:24
Quote
Did your analysis come from his book,  “Birth of Inflationary Universes”, or what was the title of the book you were referring to?

Sorry, I omitted the book title. It was:

Many Worlds in One: The Search for Other Universes

BTW, it’s on Amazon at 1P, unless you want it in Spanish, in which case it’s £659.63. It would probably be cheaper to learn English. :)

Quote
Universe: There is just one universe and it encompasses all there is, all matter, energy, everything, in one infinite and eternal presence, that had no beginning and will have no end, i.e., the universe has always existed, and has always been governed by the same set of invariant natural laws. 

Elsewhere, I have explained why I would normally use “cosmos” in this context; but “Universe” seems OK.  I would have to include one serious proviso, but this is neither the time nor place to climb into my soap box about eternity/infinity.
Title: Re: What is Nothingness?
Post by: Bogie_smiles on 26/06/2018 23:28:49
Reply #46


Sorry, I omitted the book title. It was:

Many Worlds in One: The Search for Other Universes

Quote
Universe: There is just one universe and it encompasses all there is, all matter, energy, everything, in one infinite and eternal presence, that had no beginning and will have no end, i.e., the universe has always existed, and has always been governed by the same set of invariant natural laws. 

Elsewhere, I have explained why I would normally use “cosmos” in this context; but “Universe” seems OK.  I would have to include one serious proviso, but this is neither the time nor place to climb into my soap box about eternity/infinity.

I seem to remember seeing you suggesting the use of the word “cosmos”, but don’t exactly remember why.

After the discussion of Vilenkin’s “Something from nothing” model and its apparent failure to pass the test of the “nothingness” definition, i.e., if nothingness means no space, no time, and no energy, and not even any potential for space, time, or energy, then I would conclude that no model can feature any occurrence of nothingness. Does anyone disagree?

If that is the case, then from the three explanations offered in the OP, there is only one seemingly viable explanation remaining for the existence of the universe, and that is the “Always existed” option.

Looking at the situation, since the universe does exist, then it has always existed. Therefore, a definition of the word “universe” that is consistent with a universe that has always existed seems appropriate. I’m just testing for any acceptance of my definition that does that:
Quote
Universe: There is just one universe and it encompasses all there is, all matter, energy, everything, in one infinite and eternal presence, that had no beginning and will have no end, i.e., the universe has always existed, and has always been governed by the same set of invariant natural laws.
Title: Re: What is Nothingness?
Post by: Bogie_smiles on 27/06/2018 19:24:17
Reply #47

"Universe", as defined above, is proposed as the opposite of “nothingness” from the perspective of Cosmology.

Title: Re: What is Nothingness?
Post by: Bogie_smiles on 01/07/2018 00:58:41
Reply #48

If we analyze the definition of the word “universe” as stated in reply #44 above, the question is between whether just one universe exists, or if there could be multiple universes, each separate from all others.


If there is a connection between them, or even a potential for them interacting with each other, then they fall in the category of being part of one greater universe, and the laws of nature that apply to that greater universe would apply to all the parts.


On the other hand, separate universe would mean there is no possibility of interaction between them. If the universes are completely separate from each other in space, or in different dimensions, to the extent that not even a little gravity can leak from one to another, then that scenario would qualify as multiple universes, and there could be, or likely would be different sets of natural laws that govern each separate universe.

But there is a problem with there being separate universes in regard to the proposed definition. What separates them? If it is space, then there is a potential that they can expand into each other’s space, or can move relative to teach other, giving them a potential to converge. That puts them under the definition of “universe” not multiple universes.


Further, If they are separated by another dimension that can never be detected, can we say that they even exist. I don’t think so. If we cannot say that they exist, we cannot say that there are multiple universes, and we are left with one universe, and that fits under the definition.


Conclusion, the definition of “universe” holds up when considering if there is just one universe, or if there could be multiple universes.
Title: Re: What is Nothingness?
Post by: Bogie_smiles on 01/07/2018 23:47:14
Reply #49

Quote
Universe: There is just one universe and it encompasses all there is, all matter, energy, everything, in one infinite and eternal presence, that had no beginning and will have no end, i.e., the universe has always existed, and has always been governed by the same set of invariant natural laws. 

Continuing to analyze the definition of “universe” we come to the phrase “... it encompasses all there is, all matter, energy, everything, in one infinite and eternal presence, that had no beginning and will have no end, i.e., the universe has always existed” …



The question of ”an infinite and eternal presence” has to be considered from the perspective of the alternative. If the presence of the universe is not infinite and eternal, i.e., if it has not always existed, then how do we answer the questions, what is outside of the space contained in the universe, and what was before the presence of the universe?

The answer might then logically be nothingness. However, we have already explored the definition of “nothingness” and concluded that if the universe exists, there never was nothingness. There is no provision in either definition for some kind of separation between nothingness and universe; the definitions are intended to mean that it is either complete nothingness, or the complete presence of one universe.
Title: Re: What is Nothingness?
Post by: Bogie_smiles on 03/07/2018 12:02:08
Reply #50


Quote
Universe: There is just one universe and it encompasses all there is, all matter, energy, everything, in one infinite and eternal presence, that had no beginning and will have no end, i.e., the universe has always existed, and has always been governed by the same set of invariant natural laws. 

The last phrase in the definition, “… has always been governed by the same set of invariant natural laws” means that the laws of nature don’t change. However, some laws are not yet known, some of the known laws are not yet fully understood, and the various combinations of laws that affect any given situation may be impossible to fully comprehend.

Title: Re: What is Nothingness?
Post by: Bill S on 06/07/2018 00:09:51
Quote from: Evan_au
I expect that black holes will deflect gravitons, just like they deflect photons. So maybe one day we may be able to do graviton spectroscopy by diffracting them with an array of micro-black holes?

Just checking! 

The gravitational field is scalar.  If the gravitational field is composed of gravitons, the gravitons don’t have redirection.  If this is the case; how could they be deflected?

Quote from: Evan_au
  However, real gravitons propagate away to infinity, as oscillations on the gravitational field. This is what LIGO detected. (And real photons propagate away to infinity, as oscillations on the electromagnetic field. This is what telescopes detect.)

By analogy with water waves, one might reason that individual gravitons within a gravitational wave would not propagate in the direction of the apparent motion of the wave.  Only energy would propagate in that way.

Again; how could the gravitons be deflected?
Title: Re: What is Nothingness?
Post by: Bill S on 06/07/2018 00:14:15
Quote
Universe: There is just one universe and it encompasses all there is, all matter, energy, everything, in one infinite and eternal presence, that had no beginning and will have no end….

So, the infinite past ends here, and the infinite future begins here?

 “There is something rotten in the eigenstate of Denmark”  (Adam Becker)
I can’t manage a smart remark like that, but it does seem that something is not quite as it should be.
Title: Re: What is Nothingness?
Post by: Bogie_smiles on 06/07/2018 00:28:30
Reply #53



So, the infinite past ends here, and the infinite future begins here?

 “There is something rotten in the eigenstate of Denmark”  (Adam Becker)
I can’t manage a smart remark like that, but it does seem that something is not quite as it should be.

One of my sayings comes to mind:

The universe is as it should be, and could be no other way.
Title: Re: What is Nothingness?
Post by: Bogie_smiles on 07/07/2018 22:00:57

Reply #54

Reply #50


Quote
Universe: There is just one universe and it encompasses all there is, all matter, energy, everything, in one infinite and eternal presence, that had no beginning and will have no end, i.e., the universe has always existed, and has always been governed by the same set of invariant natural laws. 

The last phrase in the definition, “… has always been governed by the same set of invariant natural laws” means that the laws of nature don’t change. However, some laws are not yet known, some of the known laws are not yet fully understood, and the various combinations of laws that affect any given situation may be impossible to fully comprehend.


The concepts of “nothingness”, and “universe”
Which are described to be opposites,
And the conclusion that the universe is governed by a set of invariant natural laws,
Many of which are as yet unknown,
Brings up the need to observe,
and the advances and challenges to our ability to observe.

We use the Scientific Method to advance science, and learn about it at  links like:
https://explorable.com/scientific-observation (https://explorable.com/scientific-observation), with starting points about the Scientific Method and our ability to observe …
(learn or teach, comment freely)
Title: Re: What is Nothingness?
Post by: Bogie_smiles on 09/07/2018 11:08:35
Reply #55

If the universe is everywhere and there is no nothingness any where, what is the smallest thing that we can observe, and what surrounds it?
Title: Re: What is Nothingness?
Post by: Bogie_smiles on 11/07/2018 11:50:34
Reply #56

Scientists are resolving images of smaller and smaller things, and they seem to be surrounded by fuzzier and fuzzier boundaries.
I found this website about learning, and searched “quantum” to see what was there:
https://quizlet.com/281471125/quantum-flash-cards/ (https://quizlet.com/281471125/quantum-flash-cards/)

Title: Re: What is Nothingness?
Post by: Jaaanosik on 11/07/2018 23:04:50
OP,
Cosmology is ...
Quote
Metaphysical philosophy ("logic") was the study of existence, causation, God, logic, forms and other abstract objects ("meta-physika" lit: "what comes after physics").[28]
...
Metaphysical philosophy has birthed formal sciences such as logic, mathematics and philosophy of science, but still includes epistemology, cosmology and others.
by this definition: https://en.wikipedia.org/wiki/Philosophy#Knowledge

You do not want to include God in this thread, fine, but cosmology is metaphysical philosophy, so it is like discussing 'faith, religion', nothing less and nothing more.
Title: Re: What is Nothingness?
Post by: Bogie_smiles on 11/07/2018 23:59:32
Reply #58


OP,
Cosmology is ...
Quote
Metaphysical philosophy ("logic") was the study of existence, causation, God, logic, forms and other abstract objects ("meta-physika" lit: "what comes after physics").[28]
...
Metaphysical philosophy has birthed formal sciences such as logic, mathematics and philosophy of science, but still includes epistemology, cosmology and others.
by this definition: https://en.wikipedia.org/wiki/Philosophy#Knowledge (https://en.wikipedia.org/wiki/Philosophy#Knowledge)

You do not want to include God in this thread, fine, but cosmology is metaphysical philosophy, so it is like discussing 'faith, religion', nothing less and nothing more.

Surely you are correct about the history of cosmology; there are age old roots, astrology, mythology, religious positions, metaphysics, as there are to the beginnings of all sciences, as supported in the Wiki:

https://en.wikipedia.org/wiki/Cosmology (https://en.wikipedia.org/wiki/Cosmology)


Cosmology (from the Greek κόσμος, kosmos "world" and -λογία, -logia "study of") is the study of the origin, evolution, and eventual fate of the universe. Physical cosmology is the scientific study of the universe's origin, its large-scale structures and dynamics, and its ultimate fate, as well as the scientific laws that govern these areas.[2]


The term cosmology was first used in English in 1656 in Thomas Blount's Glossographia,[3] and in 1731 taken up in Latin by German philosopher Christian Wolff, in Cosmologia Generalis.[4]


Religious or mythological cosmology is a body of beliefs based on mythological, religious, and esoteric literature and traditions of creation myths and eschatology.


Physical cosmology is studied by scientists, such as astronomers and physicists, as well as philosophers, such as metaphysicians, philosophers of physics, and philosophers of space and time. Because of this shared scope with philosophy, theories in physical cosmology may include both scientific and non-scientific propositions, and may depend upon assumptions that cannot be tested.


Cosmology differs from astronomy in that the former is concerned with the Universe as a whole while the latter deals with individual celestial objects. Modern physical cosmology is dominated by the Big Bang theory, which attempts to bring together observational astronomy and particle physics;[5] more specifically, a standard parameterization of the Big Bang with dark matter and dark energy, known as the Lambda-CDM model.


Theoretical astrophysicist David N. Spergel has described cosmology as a "historical science" because "when we look out in space, we look back in time" due to the finite nature of the speed of light.[6]
                                                                             
 -----------------------------

The Wiki supports your statement, and all I can say is that my stance on excluding the Supernatural from this thread recognizes the differentiation between modern hard science, which is preferred for discussion in the "hard" sciences sub-forums at the NakedScientis, and the long and varied history of man's efforts to convey the nature of things over the ages. The methodology I am invoking is the scientific method, which is well known to exclude the Supernatural and magical. Recently I mentioned that method, and provided a link for beginners who are just becoming familiar with the process.


Starting the thread with the definition of "nothingness" quickly leads to the definition of "universe", and to me, that is the basis for the discussion of today's cosmology.
Title: Re: What is Nothingness?
Post by: Jaaanosik on 12/07/2018 04:38:54
Quote
One definition of nothingness:
No space, no time, no energy, and no potential for any space, time or energy.

This definition shuts the door completely. Nothing comes from nothing.
The potentiality cannot be actualized.

If somebody wants to get something from "nothingness" then this definition of "nothingness" is not good. This definition does not yield anything.

Title: Re: What is Nothingness?
Post by: Jaaanosik on 12/07/2018 15:05:26
Quote
After the discussion of Vilenkin’s “Something from nothing” model and its apparent failure to pass the test of the “nothingness” definition, i.e., if nothingness means no space, no time, and no energy, and not even any potential for space, time, or energy, then I would conclude that no model can feature any occurrence of nothingness. Does anyone disagree?
This is good, you are coming to the conclusion that your definition of "nothingness" is not compatible with any model.
Nobody is using your definition of "nothingness".
Title: Re: What is Nothingness?
Post by: Bogie_smiles on 12/07/2018 19:11:13
Reply #61


Quote
After the discussion of Vilenkin’s “Something from nothing” model and its apparent failure to pass the test of the “nothingness” definition, i.e., if nothingness means no space, no time, and no energy, and not even any potential for space, time, or energy, then I would conclude that no model can feature any occurrence of nothingness. Does anyone disagree?
This is good, you are coming to the conclusion that your definition of "nothingness" is not compatible with any model.
Nobody is using your definition of “nothingness”.
It is not compatible with any model that invokes nothingness, or “something from nothing”. There are models that aren’t excluded by that restriction, and that don’t invoke the Supernatural.

If there never was nothingness, and therefore the universe has always existed, filling all space and time, then we have the basis for defeating that restriction.

Title: Re: What is Nothingness?
Post by: Jaaanosik on 12/07/2018 21:51:08
...

It is not compatible with any model that invokes nothingness, or “something from nothing”. There are models that aren’t excluded by that restriction, and that don’t invoke the Supernatural.

If there never was nothingness, and therefore the universe has always existed, filling all space and time, then we have the basis for defeating that restriction.

Is there a proof that the universe has always existed?
Title: Re: What is Nothingness?
Post by: Bill S on 12/07/2018 22:47:02
Quote from: Jaaanosik
This definition shuts the door completely. Nothing comes from nothing.
The potentiality cannot be actualized.

Agreed!

Quote
If somebody wants to get something from "nothingness" then this definition of "nothingness" is not good. This definition does not yield anything.

Are you suggesting that there is a way of redefining nothing to make it something, while still remaining “nothing”? 

Quote
  Is there a proof that the universe has always existed?

I believe there are some respected arguments in favour of the Universe having always existed, but proof is quite another matter.
Title: Re: What is Nothingness?
Post by: Bogie_smiles on 12/07/2018 23:00:24
Reply #64

...

It is not compatible with any model that invokes nothingness, or “something from nothing”. There are models that aren’t excluded by that restriction, and that don’t invoke the Supernatural.

If there never was nothingness, and therefore the universe has always existed, filling all space and time, then we have the basis for defeating that restriction.


Is there a proof that the universe has always existed?
Quote from: Jaaanosik
This definition shuts the door completely. Nothing comes from nothing.
The potentiality cannot be actualized.


Agreed!


Quote
If somebody wants to get something from "nothingness" then this definition of "nothingness" is not good. This definition does not yield anything.


Are you suggesting that there is a way of redefining nothing to make it something, while still remaining “nothing”? 


Quote
  Is there a proof that the universe has always existed?


I believe there are some respected arguments in favour of the Universe having always existed, but proof is quite another matter.


Is there proof that there was a beginning?


The MIT Technology Review had an article on the topic in April 24, 2012:


https://www.technologyreview.com/s/427722/mathematics-of-eternity-prove-the-universe-must-have-had-a-beginning/ (https://www.technologyreview.com/s/427722/mathematics-of-eternity-prove-the-universe-must-have-had-a-beginning/)


A View from Emerging Technology from the arXiv (https://www.technologyreview.com/profile/emerging-technology-from-the-arxiv/)
Mathematics of Eternity Prove The Universe Must Have Had A Beginning
Cosmologists use the mathematical properties of eternity to show that although universe may last forever, it must have had a beginning
April 24, 2012

The Big Bang has become part of popular culture since the phrase was coined by the maverick physicist Fred Hoyle in the 1940s. That’s hardly surprising for an event that represents the ultimate birth of everything.

However, Hoyle much preferred a different model of the cosmos: a steady state universe with no beginning or end, that stretches infinitely into the past and the future. That idea never really took off.

In recent years, however, cosmologists have begun to study a number of new ideas that have similar properties. Curiously, these ideas are not necessarily at odds with the notion of a Big Bang.

For instance, one idea is that the universe is cyclical with big bangs followed by big crunches followed by big bangs in an infinite cycle.

Another is the notion of eternal inflation in which different parts of the universe expand and contract at different rates. These regions can be thought of as different universes in a giant multiverse.

So although we seem to live in an inflating cosmos,  other universes may be very different. And while our universe may look as if it has a beginning, the multiverse need not have a beginning.

Then there is the idea of an emergent universe which exists as a kind of seed for eternity and then suddenly expands.

So these modern cosmologies suggest that the observational evidence of an expanding universe is consistent with a cosmos with no beginning or end. That may be set to change.

Today, Audrey Mithani and Alexander Vilenkin at Tufts University in Massachusetts say that these models are mathematically incompatible with an eternal past. Indeed, their analysis suggests that these three models of the universe must have had a beginning too.

Their argument focuses on the mathematical properties of eternity–a universe with no beginning and no end. Such a universe must contain trajectories that stretch infinitely into the past.

However, Mithani and Vilenkin point to a proof dating from 2003 that these kind of past trajectories cannot be infinite if they are part of a universe that expands in a specific way.

They go on to show that cyclical universes and universes of eternal inflation both expand in this way. So they cannot be eternal in the past and must therefore have had a beginning. “Although inflation may be eternal in the future, it cannot be extended indefinitely to the past,” they say.

They treat the emergent model of the universe differently, showing that although it may seem stable from a classical point of view, it is unstable from a quantum mechanical point of view. “A simple emergent universe model…cannot escape quantum collapse,” they say.

The conclusion is inescapable. “None of these scenarios can actually be past-eternal,” say Mithani and Vilenkin.

Since the observational evidence is that our universe is expanding, then it must also have been born in the past. A profound conclusion (albeit the same one that lead to the idea of the big bang in the first place).
 
Ref: arxiv.org/abs/1204.4658: Did The Universe Have A Beginning?

———————————
Title: Re: What is Nothingness?
Post by: Jaaanosik on 12/07/2018 23:59:28
Quote
Are you suggesting that there is a way of redefining nothing to make it something, while still remaining “nothing”? 

What do you mean by 'still remaining "nothing"'?
If it is supposed to be still your definition of nothing with your "units of measure" of nothing (no space, no time, no energy, and no potential for any space, time or energy) then I already said it cannot be done. You closed the door on "nothingness".

... but if we define our space-time as something and everything else what is outside of our space-time as nothing then it is going to work.

Title: Re: What is Nothingness?
Post by: Jaaanosik on 13/07/2018 00:02:33
Hmmm, actually thinking about it, I can 'clarify' your definition of nothingness and it is going to work.
Later...
Title: Re: What is Nothingness?
Post by: Bogie_smiles on 13/07/2018 00:39:52

Reply #67

Hmmm, actually thinking about it, I can 'clarify' your definition of nothingness and it is going to work.
Later...
I'm skeptical that there can be a clarification that doesn't make the definition loose something in translation, :)
Title: Re: What is Nothingness?
Post by: Bogie_smiles on 16/07/2018 12:09:56
Reply #68




Is there a proof that the universe has always existed?
To which I replied by asking, “Is there a proof that there was a beginning?”


That question came from the link I provided in Reply #64 to the article in The MIT Technology Review on the topic in April 24, 2012. See reply #64 above for the content of the article. MIT’s lead in to the article said:
“Cosmologists use the mathematical properties of eternity to show that although [the] universe may last forever, it must have had a beginning”.


Audrey Mithani and Alexander Vilenkin at Tufts University in Massachusetts go on to conclude that the “always existed” models that they discuss are mathematically incompatible with an eternal past. Indeed, their analysis suggests that, “[those] models of the universe must have had a beginning too.”


They reason that, “Since the observational evidence is that our universe is expanding, then it must also have been born in the past. A profound conclusion (albeit the same one that lead to the idea of the big bang in the first place).”


There is an obvious flaw to that reasoning if you consider what we can and can’t see of our known universe. We can see a large segment showing the history of our big bang environment, but no one claims we can see back to the beginning, or that what we can see indicates that is all there is. We see only the observable segment of the greater universe, with strong evidence that a big bang event occurred in the past. There is sufficient evidence of a big bang event in the “Hubble” red shift data (link: http://www.pbs.org/wgbh/aso/databank/entries/dp29hu.html (http://www.pbs.org/wgbh/aso/databank/entries/dp29hu.html)).



That view is giving us a look back of about 14 billion years, back to the time when our "big bang" event was very young, and from a period when photon light was beginning to be emitted from the newly forming atoms as the extreme hot high density of the big bang expanded and cooled.


However, Mithani and Vilenkin are making a judgement call if you accept the law of cause and effect. (Link:  https://en.wikipedia.org/wiki/Causality), and there is sound reasoning that such an event would have an event horizon that limits our ability to look back to the big bang itself. Reaching the conclusion that the expansion we can observe can be back tracked to the beginning of the universe is not the same thing as saying, “Since the observational evidence is that our universe is expanding …”. The observable universe is expanding, but that is only the part that we can observe.


The distinction between our big bang being born in the past, and the universe being born in the past seems to equate the two as being “one and the same” buy they are not. That conclusion is flawed, simply based on the reasoning of cause and effect. It is much more prudent to conclude that our big bang happened as a result of preconditions that were present before our big bang event, and not from the very birth of the universe out of nothingness.
Title: Re: What is Nothingness?
Post by: syhprum on 16/07/2018 13:11:24
There is a small village near me on the liver Loose also called loose it has a women's  institute appropriately called the Loose women's institute which would lose its amusing title if everyone spelt lose as loose
Title: Re: What is Nothingness?
Post by: Bogie_smiles on 16/07/2018 13:27:40
North Yorkshire, I presume.
Title: Re: What is Nothingness?
Post by: Jaaanosik on 16/07/2018 15:44:26
When electron emits a photon in the hydrogen atom where is photon coming from?
Is it somehow related to nothingness? Quantum fluctuations?
What is electron? Positron and electron collision leads to two gamma ray photons.

Could we imagine an electron as a bunch of tiny photons 'trapped' in a rotation?
Photons translational kinetic energy is 'trapped' into the rotational kinetic energy?
When the collision between electron and positron happens the bunch of tiny photons change the rotational kinetic energy to translational kinetic energy and the bunch flies away as what we observe like a gamma ray photon.

So what is photon? Let us imagine a 3D matrix of quantum fluctuations.
The fluctuations are like Newton cradle with elastic interactions between the points:
(https://theelectromagneticnatureofthings.com/img/i9/newton_cradle.png)
Photon is an energy traveling through the matrix of quantum fluctuations like it is a case in the Newton cradle.

This is very simple world view concept of our space-time fabric.
Going back to your definition of nothingness. Do you consider quantum fluctuations as something that has potential for energy or that quantum fluctuations are already energy? In other words energy exists only when it propagates through the quantum fluctuations - photons or existence of quantum fluctuations is energy already.
Title: Re: What is Nothingness?
Post by: Bill S on 16/07/2018 16:12:57
Quote from: Jaaanosik
If somebody wants to get something from "nothingness" then this definition of "nothingness" is not good. This definition does not yield anything.

Quote from: Bill
Are you suggesting that there is a way of redefining nothing to make it something, while still remaining “nothing”? 

Quote
What do you mean by 'still remaining "nothing"'?

Your quote suggests that you think there is more than one definition of “nothing”.  As I see it, there is only one definition: “nothing”.  I was asking if you could give an example of another definition that was not “something”.

It gets convoluted when you start “messing” with nothing; doesn’t it? :)
Title: Re: What is Nothingness?
Post by: Jaaanosik on 17/07/2018 00:35:09
I have to clarify I still have the OP definition in mind:
Quote
No space, no time, no energy, and no potential for any space, time or energy.

When we consider the quantum fluctuations as something, then where do they come from?
It seems like 'nothing' from within our space-time dimensions. That 'nothingness' outside of our space-time has no potential for energy that we can observe within our space-time therefore I would say it qualifies as 'nothingness' from our space-time point of view.
Title: Re: What is Nothingness?
Post by: Bogie_smiles on 17/07/2018 01:37:23
Reply #74


I have to clarify I still have the OP definition in mind:
Quote
No space, no time, no energy, and no potential for any space, time or energy.

When we consider the quantum fluctuations as something, then where do they come from?
It seems like 'nothing' from within our space-time dimensions. That 'nothingness' outside of our space-time has no potential for energy that we can observe within our space-time therefore I would say it qualifies as 'nothingness' from our space-time point of view.

You raise questions, and propose ideas about some aspects of nature that are addressed by different scientific disciplines, pertain to different theories of cosmology, and that aren’t expected to work together.

I do not agree that you have a case of nothingness yet either, and note that the intention of the definition in the OP is to preclude the possibility that there can ever be nothingness, given the fact that the universe exists.

Here are the Wikis on spacetime and quantum fluctuations for review:
https://en.wikipedia.org/wiki/Spacetime (https://en.wikipedia.org/wiki/Spacetime)

https://en.wikipedia.org/wiki/Quantum_fluctuation (https://en.wikipedia.org/wiki/Quantum_fluctuation)

Title: Re: What is Nothingness?
Post by: Jaaanosik on 17/07/2018 02:18:14

I have to clarify I still have the OP definition in mind:
Quote
No space, no time, no energy, and no potential for any space, time or energy.


When we consider the quantum fluctuations as something, then where do they come from?
It seems like 'nothing' from within our space-time dimensions. That 'nothingness' outside of our space-time has no potential for energy that we can observe within our space-time therefore I would say it qualifies as 'nothingness' from our space-time point of view.


You raise questions, and propose ideas about some aspects of nature that are addressed by different scientific disciplines, pertain to different theories of cosmology, and that aren’t expected to work together.

I do not agree that you have a case of nothingness yet either, and note that the intention of the definition in the OP is to preclude the possibility that there can ever be nothingness, given the fact that the universe exists.

Here are the Wikis on spacetime and quantum fluctuations for review:
https://en.wikipedia.org/wiki/Spacetime (https://en.wikipedia.org/wiki/Spacetime)

https://en.wikipedia.org/wiki/Quantum_fluctuation (https://en.wikipedia.org/wiki/Quantum_fluctuation)


Where do quantum fluctuations come from?
Title: Re: What is Nothingness?
Post by: Bogie_smiles on 17/07/2018 21:50:15
Reply #76



Where do quantum fluctuations come from?

The link to “quantum fluctuation” answers that question, from the perspective of quantum physics, also known as quantum mechanics:
Quantum fluctuations of a field

“A quantum fluctuation is the temporary appearance of energetic particles out of empty space (see my note below), as allowed by the uncertainty principle (https://en.wikipedia.org/wiki/Uncertainty_principle). The uncertainty principle states that for a pair of conjugate variables such as position/momentum or energy/time, it is impossible to have a precisely determined value of each member of the pair at the same time.”

This description may not resonate with you if you are not familiar with the basics of quantum mechanics. https://en.wikipedia.org/wiki/Quantum_mechanics (https://en.wikipedia.org/wiki/Quantum_mechanics)

Note: In regard to your search for some example of nothingness in the universe, the use of the phrase “empty space” does not denote a condition of nothingness. Quantum mechanics is incomplete, and the origin of the universe is a concern, as indicated by the statement in the QM link:

“Quantum fluctuations may have been very important in the origin of the structure of the universe: according to the model of expansive inflation the ones that existed when inflation began were amplified and formed the seed of all current observed structure.”

You can tell from that quote that when they mention the “origin of the structure of the universe”, there were quantum fluctuations in existence when the inflation began, i.e., a precondition to inflation, but not a state of nothingness.

The explanation goes on: “In the modern view, energy is always conserved, but because the particle number operator does not commute with a field's Hamiltonian or energy operator, the field's lowest-energy or ground state, often called the vacuum state, is not, as one might expect from that name, a state with no particles, but rather a quantum superposition of particle number eigenstates with 0, 1, 2...etc. particles.”

Title: Re: What is Nothingness?
Post by: Jaaanosik on 17/07/2018 23:14:07

Where do quantum fluctuations come from?

The link to “quantum fluctuation” answers that question, from the perspective of quantum physics, also known as quantum mechanics:
Quantum fluctuations of a field

“A quantum fluctuation is the temporary appearance of energetic particles out of empty space (see my note below), as allowed by the uncertainty principle (https://en.wikipedia.org/wiki/Uncertainty_principle). The uncertainty principle states that for a pair of conjugate variables such as position/momentum or energy/time, it is impossible to have a precisely determined value of each member of the pair at the same time.”

This description may not resonate with you if you are not familiar with the basics of quantum mechanics. https://en.wikipedia.org/wiki/Quantum_mechanics (https://en.wikipedia.org/wiki/Quantum_mechanics)

Note: In regard to your search for some example of nothingness in the universe, the use of the phrase “empty space” does not denote a condition of nothingness. Quantum mechanics is incomplete, and the origin of the universe is a concern, as indicated by the statement in the QM link:

“Quantum fluctuations may have been very important in the origin of the structure of the universe: according to the model of expansive inflation the ones that existed when inflation began were amplified and formed the seed of all current observed structure.”

You can tell from that quote that when they mention the “origin of the structure of the universe”, there were quantum fluctuations in existence when the inflation began, i.e., a precondition to inflation, but not a state of nothingness.

The explanation goes on: “In the modern view, energy is always conserved, but because the particle number operator does not commute with a field's Hamiltonian or energy operator, the field's lowest-energy or ground state, often called the vacuum state, is not, as one might expect from that name, a state with no particles, but rather a quantum superposition of particle number eigenstates with 0, 1, 2...etc. particles.”


Everything that you posted has to be taken at faith value. There are lots of unknowns.
We have 'nothing'!
There is no experiment to show where the quantum fluctuations come from!
We have nothing to show what is the cause of their existence.

If QM is incomplete how we can claim we know what is empty space?
We do not know what empty space is, again we have 'nothing'!

Quote
Quantum fluctuations of a field
What field? Where does it come from? Is this chicken egg problem? What comes first?
We have 'nothing'!

As I said, everything you posted is full of holes.
Are we going to stay truthful to logic? Are we going to be honest with ourselves?
Title: Re: What is Nothingness?
Post by: Bogie_smiles on 17/07/2018 23:37:26
Reply #78


Everything that you posted has to be taken at faith value. There are lots of unknowns.
It is true. The body of scientific knowledge includes known science, and as yet unknown science, and it is the intention of the scientific community, using the scientific method, to advance the body of scientific knowledge.
Quote
We have 'nothing'!
There is no experiment to show where the quantum fluctuations come from!
We have nothing to show what is the cause of their existence.

If QM is incomplete how we can claim we know what is empty space?
We do not know what empty space is, again we have 'nothing'!

Quantum fluctuations of a field …
What field? Where does it come from? Is this chicken egg problem? What comes first?
We have 'nothing'!

As I said, everything you posted is full of holes.
Yes, those are criticisms that can be tossed out if you think that the scientific community is saying we “know” those things. However, one of the pillars of science is called “the tentativeness of science”, meaning that there are generally accepted theories, and every theory is subject to being superseded by new discoveries and new theories that are a better fit to the body of scientific knowledge. The advance of science is a process, and the state of our advance is encompassed in what I referred to as the “body of scientific knowledge”. It doesn’t mean everything is agreed on, or that there aren’t competing theories, but the process is ongoing, and the advance of our knowledge remains a common objective.
Quote

Are we going to stay truthful to logic? Are we going to be honest with ourselves?
How about both.  Logic however, is not universal, it is what drives the consensus; when the scientific community agrees on a given theory over other competing theories, it is not unanimous, it is subject to tentativeness, and it exists with the expectation of ongoing discovery and change.

If you are truthful and logical, you will be able to be both a skeptic, and a science enthusiast.

Title: Re: What is Nothingness?
Post by: Jaaanosik on 17/07/2018 23:41:39
Everything that you posted has to be taken at faith value. There are lots of unknowns.
It is true. The body of scientific knowledge includes known science, and as yet unknown science, and it is the intention of the scientific community, using the scientific method, to advance the body of scientific knowledge.
Quote
We have 'nothing'!
There is no experiment to show where the quantum fluctuations come from!
We have nothing to show what is the cause of their existence.

If QM is incomplete how we can claim we know what is empty space?
We do not know what empty space is, again we have 'nothing'!

Quantum fluctuations of a field …
What field? Where does it come from? Is this chicken egg problem? What comes first?
We have 'nothing'!

As I said, everything you posted is full of holes.
Yes, those are criticisms that can be tossed out if you think that the scientific community is saying we “know” those things. However, one of the pillars of science is called “the tentativeness of science”, meaning that there are generally accepted theories, and every theory is subject to being superseded by new discoveries and new theories that are a better fit to the body of scientific knowledge. The advance of science is a process, and the state of our advance is encompassed in what I referred to as the “body of scientific knowledge”. It doesn’t mean everything is agreed on, or that there aren’t competing theories, but the process is ongoing, and the advance of our knowledge remains a common objective.
Quote

Are we going to stay truthful to logic? Are we going to be honest with ourselves?
How about both.  Logic however, is not universal, it is what drives the consensus; when the scientific community agrees on a given theory over other competing theories, it is not unanimous, it is subject to tentativeness, and it exists with the expectation of ongoing discovery and change.

If you are truthful and logical, you will be able to be both a skeptic, and a science enthusiast.


Are you saying, in other words, yeap, we have nothing? :)
Title: Re: What is Nothingness?
Post by: Bogie_smiles on 17/07/2018 23:49:16
Reply #80



Are you saying, in other words, yeap, we have nothing? :)
LOL, not exactly. We have many generally accepted scientific observations, and a growing body of knowledge that represents the efforts of the scientific community to understand them.

https://en.wikipedia.org/wiki/List_of_experiments (https://en.wikipedia.org/wiki/List_of_experiments)

https://en.wikipedia.org/wiki/Category:Science-related_lists (https://en.wikipedia.org/wiki/Category:Science-related_lists)



Title: Re: What is Nothingness?
Post by: Jaaanosik on 18/07/2018 00:01:11

Are you saying, in other words, yeap, we have nothing? :)
LOL, not exactly. We have many generally accepted scientific observations, and a growing body of knowledge that represents the efforts of the scientific community to understand them.

https://en.wikipedia.org/wiki/List_of_experiments (https://en.wikipedia.org/wiki/List_of_experiments)

 
We have no experiment to show where the quantum fluctuations come from.
Agreed?
What is your take on causality in physics?
Title: Re: What is Nothingness?
Post by: Bogie_smiles on 18/07/2018 00:07:52

Reply #82


We have no experiment to show where the quantum fluctuations come from.
Agreed?
The understanding of quantum fluctuations is theoretical, and I would agree that from the perspective of our skeptical natures, we don't have any such experiments.
Quote
What is your take on causality in physics?
I'm a cause and effect type of individual. The universe, according to the definition that I have offered, has always existed, and so there was no first cause.

Why don't you take a minute and let me know where you are coming from in this exchange.
Title: Re: What is Nothingness?
Post by: Jaaanosik on 18/07/2018 00:31:01

We have no experiment to show where the quantum fluctuations come from.
Agreed?
The understanding of quantum fluctuations is theoretical, and I would agree that from the perspective of our skeptical natures, we don't have any such experiments.
Quote
What is your take on causality in physics?
I'm a cause and effect type of individual. The universe, according to the definition that I have offered, has always existed, and so there was no first cause.

Why don't you take a minute and let me know where you are coming from in this exchange.

My post #71 is essential to understand my position.
I see quantum fluctuations as fundamental foundation for existence of universe. There would be no photons, no matter without quantum fluctuations.
If quantum fluctuations exist then they have a cause therefore there is some first cause.

Having said that I am fine to admit that the first cause is not 'detectable' to us physically, from within our space-time, so it appears to us as 'nothing'.
Title: Re: What is Nothingness?
Post by: Bogie_smiles on 18/07/2018 01:12:16
Reply #84


My post #71 is essential to understand my position.
I see quantum fluctuations as fundamental foundation for existence of universe. There would be no photons, no matter without quantum fluctuations.
If quantum fluctuations exist then they have a cause therefore there is some first cause.

Having said that I am fine to admit that the first cause is not 'detectable' to us physically, from within our space-time, so it appears to us as 'nothing'.
Are you saying that the quantum fluctuations had to precede the existence of the photons and fundamental particles that currently make up the universe?

If we were to follow that reasoning, within that perspective I could imagine some possibilities:


I would suggest that the quantum fluctuations could have always existed as an ongoing process of random fluctuation across all space, and would have always participated in the process of particle formation. A co-existence of quantum fluctuations which are ongoing, and of the particles that currently make up the universe.

Does that seem logical to you?
Title: Re: What is Nothingness?
Post by: Jaaanosik on 18/07/2018 01:58:51
My post #71 is essential to understand my position.
I see quantum fluctuations as fundamental foundation for existence of universe. There would be no photons, no matter without quantum fluctuations.
If quantum fluctuations exist then they have a cause therefore there is some first cause.

Having said that I am fine to admit that the first cause is not 'detectable' to us physically, from within our space-time, so it appears to us as 'nothing'.
Are you saying that the quantum fluctuations had to precede the existence of the photons and fundamental particles that currently make up the universe?

If we were to follow that reasoning, within that perspective I could imagine some possibilities:


I would suggest that the quantum fluctuations could have always existed as an ongoing process of random fluctuation across all space, and would have always participated in the process of particle formation. A co-existence of quantum fluctuations which are ongoing, and of the particles that currently make up the universe.

Does that seem logical to you?

Now I am going to give you my testimony.
You have two options, accept it as true statement or rejected it.
If you rejected it, you would consider me as a liar, but I have no reason to lie about it.

The testimony - what you said is almost exactly what I had in mind. This is logical.
I said almost because this has a continuation though. The quantum fluctuations are something and therefore they must have the first cause. This is the starting point.

The next question is where the photons come from? What caused them to be created?
Title: Re: What is Nothingness?
Post by: Bogie_smiles on 18/07/2018 02:08:03
Reply #86



Now I am going to give you my testimony.
You have two options, accept it as true statement or rejected it.
If you rejected it, you would consider me as a liar, but I have no reason to lie about it.

The testimony - what you said is almost exactly what I had in mind. This is logical.
I said almost because this has a continuation though. The quantum fluctuations are something and therefore they must have the first cause. This is the starting point.

The next question is where the photons come from? What caused them to be created?

I believe you :)

That brings us to the point where I have a suggestion. Start a thread in New Theories, and let's carry on with this session over there. The reason, this sub-forum is more for hard science, and we are getting into speculative ideas that I prefer not to get into in this thread. Any objections to that?
Title: Re: What is Nothingness?
Post by: Jaaanosik on 18/07/2018 03:03:43

Now I am going to give you my testimony.
You have two options, accept it as true statement or rejected it.
If you rejected it, you would consider me as a liar, but I have no reason to lie about it.

The testimony - what you said is almost exactly what I had in mind. This is logical.
I said almost because this has a continuation though. The quantum fluctuations are something and therefore they must have the first cause. This is the starting point.

The next question is where the photons come from? What caused them to be created?

I believe you :)

That brings us to the point where I have a suggestion. Start a thread in New Theories, and let's carry on with this session over there. The reason, this sub-forum is more for hard science, and we are getting into speculative ideas that I prefer not to get into in this thread. Any objections to that?
Yes, I object. :)

I object, because I am presenting two 'tentative pillars':
1. Where the quantum fluctuations come from - already explained.
2. Where the photons come from - I did not say that but I am going to say it now. I see the same first cause giving energy to photons.
These are two building blocks that are 'tentative' the rest is physics.

Photon creating mass particle? It is physics and if we believe this testimony: https://en.wikipedia.org/wiki/Positron
Quote
Positrons may be generated by positron emission radioactive decay (through weak interactions), or by pair production from a sufficiently energetic photon which is interacting with an atom in a material.
... then it is already proven by experiment.
Nothing else is needed. We have our universe.

Now your turn to show any other cosmological 'theory' that is simpler. 'Theory' that has less starting building blocks that are less 'tentative'.
I am done. :)
I'll just comment if I find something interesting here.
Title: Re: What is Nothingness?
Post by: Bogie_smiles on 18/07/2018 11:26:22
Reply #88

Jaaanosik Reply #87

Yes, I object. :)

I object, because I am presenting two 'tentative pillars':
1. Where the quantum fluctuations come from - already explained.
2. Where the photons come from - I did not say that but I am going to say it now. I see the same first cause giving energy to photons.
These are two building blocks that are 'tentative' the rest is physics.

Photon creating mass particle? It is physics and if we believe this testimony: https://en.wikipedia.org/wiki/Positron
Quote
Positrons may be generated by positron emission radioactive decay (through weak interactions), or by pair production from a sufficiently energetic photon which is interacting with an atom in a material.

... then it is already proven by experiment.
Nothing else is needed. We have our universe.

Now your turn to show any other cosmological 'theory' that is simpler. 'Theory' that has less starting building blocks that are less 'tentative'.
I am done. :)
I'll just comment if I find something interesting here.

Good job. For me to add anything of interest that isn’t already under consideration by the scientific community, and documented in Wiki, lol, would be speculation on my part.

Earlier in the thread there was discussion about how the first cause is often not addressed in various theories and models of the universe because we just don’t know yet. Additionally, there is the fact that the Supernatural is not considered scientific from the perspective of the scientific method, and so doesn’t qualify as a scientific first cause, and thus violates causality.

Your two tentative pillars and the Wiki on positrons lend support to your conclusion that, “Nothing else is needed. We have our universe”, but if your model is quantum mechanics, the consensus is that the theory is incomplete and not yet considered the consensus model of the universe. If your first cause is “always existed” then quantum mechanics doesn’t go there, and you have put words into the existing body of knowledge. Those words put your revision of QM into the speculative New Theories category, IMHO.
Title: Re: What is Nothingness?
Post by: Jaaanosik on 18/07/2018 14:46:57
Reply #88

Jaaanosik Reply #87

Yes, I object. :)

I object, because I am presenting two 'tentative pillars':
1. Where the quantum fluctuations come from - already explained.
2. Where the photons come from - I did not say that but I am going to say it now. I see the same first cause giving energy to photons.
These are two building blocks that are 'tentative' the rest is physics.

Photon creating mass particle? It is physics and if we believe this testimony: https://en.wikipedia.org/wiki/Positron
Quote
Positrons may be generated by positron emission radioactive decay (through weak interactions), or by pair production from a sufficiently energetic photon which is interacting with an atom in a material.

... then it is already proven by experiment.
Nothing else is needed. We have our universe.

Now your turn to show any other cosmological 'theory' that is simpler. 'Theory' that has less starting building blocks that are less 'tentative'.
I am done. :)
I'll just comment if I find something interesting here.

Good job. For me to add anything of interest that isn’t already under consideration by the scientific community, and documented in Wiki, lol, would be speculation on my part.

It's OK to speculate. What's the big deal? :)
Cosmology is philosophy and logic.
Quote
Earlier in the thread there was discussion about how the first cause is often not addressed in various theories and models of the universe because we just don’t know yet. Additionally, there is the fact that the Supernatural is not considered scientific from the perspective of the scientific method, and so doesn’t qualify as a scientific first cause, and thus violates causality.


I disagree that it violates the causality. The first cause is a part and it has been a part of the philosophy for centuries.
It is part of the logic.
Quote

Your two tentative pillars and the Wiki on positrons lend support to your conclusion that, “Nothing else is needed. We have our universe”, but if your model is quantum mechanics, the consensus is that the theory is incomplete and not yet considered the consensus model of the universe. If your first cause is “always existed” then quantum mechanics doesn’t go there, and you have put words into the existing body of knowledge. Those words put your revision of QM into the speculative New Theories category, IMHO.


The QM comes after the first two building blocks. It appears to me it is complete in this sense.

The other hypothetical models have issues.
QM alone has an issue of - 'Quantum fluctuations of a field' - what field? Where does it come from?
Big Bang - where the singularity comes from? What's the cause?
The universe existed all the time - how? It breaks causality.

Yes, we can call what I presented as 'the new speculative hypothetical model' - no problem. This is not theory. A theory can come from a proven hypothetical model. Cosmological hypothetical models cannot be proven therefore they stay hypothetical models. There is no theory here.

This 'new speculative hypothetical model' appears to be better than anything out there based on the Occam's razor.

Title: Re: What is Nothingness?
Post by: Bill S on 18/07/2018 15:10:48
I don’t have time to read all the contributions since my last post.  I had a quick look to see if there was a straightforward response to my question in #72.  Perhaps I missed it.

From the quantity of, undoubtedly interesting posts, it would seem that the “identity” of “nothing” is far more complex than I ever suspected.  Perhaps it has something to do with the fact that there are experts who seem bent on convincing us that the Universe emerged from nothing.

I have yet to find anyone who can give an example of something from nothing.  The nearest anyone has come was JP, for whom I have great respect, and to whom I owe what understanding I have of a number of things; but even he “fell at the last hurdle” on this one.   

Possibly there is no answer that makes any sense in our 3+1D Universe, but if that’s the case, there is certainly a reluctance to admit it.  It’s a bit like asking for a physical example of “A point on a line [that] has no length”.
Title: Re: What is Nothingness?
Post by: Bogie_smiles on 18/07/2018 16:03:57

Reply #91

It's OK to speculate. What's the big deal? :)
Cosmology is philosophy and logic.
Quote from: Bogie
Earlier in the thread there was discussion about how the first cause is often not addressed in various theories and models of the universe because we just don’t know yet. Additionally, there is the fact that the Supernatural is not considered scientific from the perspective of the scientific method, and so doesn’t qualify as a scientific first cause, and thus violates causality.


I disagree that it violates the causality. The first cause is a part and it has been a part of the philosophy for centuries.
It is part of the logic.
Quote from: Bogie

Your two tentative pillars and the Wiki on positrons lend support to your conclusion that, “Nothing else is needed. We have our universe”, but if your model is quantum mechanics, the consensus is that the theory is incomplete and not yet considered the consensus model of the universe. If your first cause is “always existed” then quantum mechanics doesn’t go there, and you have put words into the existing body of knowledge. Those words put your revision of QM into the speculative New Theories category, IMHO.


The QM comes after the first two building blocks. It appears to me it is complete in this sense.

The other hypothetical models have issues.
QM alone has an issue of - 'Quantum fluctuations of a field' - what field? Where does it come from?
Big Bang - where the singularity comes from? What's the cause?
The universe existed all the time - how? It breaks causality.

Yes, we can call what I presented as 'the new speculative hypothetical model' - no problem. This is not theory. A theory can come from a proven hypothetical model. Cosmological hypothetical models cannot be proven therefore they stay hypothetical models. There is no theory here.

This 'new speculative hypothetical model' appears to be better than anything out there based on the Occam's razor.
All right, let’s let that stand and see if it gets addressed with any supporting or opposing arguments from the membership.
Title: Re: What is Nothingness?
Post by: Bogie_smiles on 18/07/2018 16:15:08
Reply #92


I don’t have time to read all the contributions since my last post.  I had a quick look to see if there was a straightforward response to my question in #72.  Perhaps I missed it.

From the quantity of, undoubtedly interesting posts, it would seem that the “identity” of “nothing” is far more complex than I ever suspected.  Perhaps it has something to do with the fact that there are experts who seem bent on convincing us that the Universe emerged from nothing.

I have yet to find anyone who can give an example of something from nothing.  The nearest anyone has come was JP, for whom I have great respect, and to whom I owe what understanding I have of a number of things; but even he “fell at the last hurdle” on this one.   

Possibly there is no answer that makes any sense in our 3+1D Universe, but if that’s the case, there is certainly a reluctance to admit it.  It’s a bit like asking for a physical example of “A point on a line [that] has no length”.
I don’t think there is a reluctance to admit that the current consensus view of cosmology, which I think can be referred to as Big Bang Theory with Inflation, and consists of General Relativity and Inflationary Theory, is presented as reality, or as the final chapter. It is the generally accepted “best yet” cosmological model, but I don’t see it being defended as if there wasn’t ongoing work to get a new consensus on a different model.  Certainly there is a concentrated effort to put together a unifying model that includes a quantum solution to gravity.

The name of the game for layman science enthusiasts is to wait as the scientific community makes their advances; I call it the Big Wait, :)
Title: Re: What is Nothingness?
Post by: yor_on on 18/07/2018 16:41:14
'Nothingness' is the fourdimensional universe we live in, speaking mainstreamly :)
It could be called a vacuum with fluctuations, but  'vacuum' contain 'possibilities', some of which are pretty long-lived. Why that is seems to be about the 'time' it takes for a 'universe' to exist and 'cool down', from some extremely hot state at its beginning. The rest is about transformations and entropy as I get it :)

there's a lot of vacuum around.
Title: Re: What is Nothingness?
Post by: yor_on on 18/07/2018 16:58:09
There are two very good questions you can ask
What makes a vacuum
Why is there atoms
=

you might find those two interesting
https://ned.ipac.caltech.edu/level5/Guth/Guth3.html

And then about pressure
https://physics.stackexchange.com/questions/3688/why-does-pressure-act-as-a-source-for-the-gravitational-field
Title: Re: What is Nothingness?
Post by: Bogie_smiles on 18/07/2018 17:32:13
Reply #95


'Nothingness' is the fourdimensional universe we live in, speaking mainstreamly :)
It could be called a vacuum with fluctuations, but  'vacuum' contain 'possibilities', some of which are pretty long-lived. Why that is seems to be about the 'time' it takes for a 'universe' to exist and 'cool down', from some extremely hot state at its beginning. The rest is about transformations and entropy as I get it :)


there's a lot of vacuum around.






To be sure, the vacuum of space is almost everywhere :)


I would like to address your statement, “[It] seems to be about the 'time' it takes for a 'universe' to exist and 'cool down', from some extremely hot state at it's beginning”. The phrase “at it’s beginning” refers to the our implied big bang event …


Now, if the definitions of “nothingness” and of “universe” that I have offered which are as follows:


One definition of nothingness (from the OP):
No space, no time, no energy, and no potential for any space, time or energy.


Definition of Universe from reply #44: There is just one universe and it encompasses all there is, all matter, energy, everything, in one infinite and eternal presence, that had no beginning and will have no end, i.e., the universe has always existed, and has always been governed by the same set of invariant natural laws.




… then the Big Bang was not the beginning of the universe. It would be an event that would have had preconditions within a greater universe, which has always existed. Can you go with that or do you have changes that you would make to my definitions of nothingness and universe?

Title: Re: What is Nothingness?
Post by: yor_on on 18/07/2018 17:50:54
I don't know Bogie.
And as usual I kept adding to my last reply before seeing yours :)
But if you look at my reply before this you will see a link to Alan Guth in where he discuss the initial conditions of a inflation.
Title: Re: What is Nothingness?
Post by: yor_on on 18/07/2018 17:59:56
If you think of it most of the current knowledge points to a very high energy density in the beginning. The way stars made us is a direct result from transformations needing heat aka 'kinetic motion'. It doesn't really discuss a geometry although we presume there to be needed some sort of confinement for it to exist, well, at least I do :)

But without 'particles' transferring this 'heat' those initial conditions becomes one of pressure, which then links to Einsteins stress energy tensor. It's interesting

And a very high energy density is also about heat.
=

And if we want to add to that we can consider the homogeneous isotropic universe we find around us. A 'Big Bang' whatever that means is not a centered source, situated at some 'middle of the universe'. In a way it never stopped as every 'point' of this universe still expands. No center to it, and the proof of that lies in astronomical observations of 'early light', being all around us. I can't see any other way to understand it myself.
Title: Re: What is Nothingness?
Post by: Bill S on 18/07/2018 21:21:37
Quote from: Yor_on
If you think of it most of the current knowledge points to a very high energy density in the beginning.

Presumably, you mean the beginning of our "observable" Universe; not the beginning of everything; before which there would have been "nothing".
Title: Re: What is Nothingness?
Post by: Bogie_smiles on 18/07/2018 21:34:20
Reply #99
There are two very good questions you can ask
What makes a vacuum
Why is there atoms
=

you might find those two interesting
https://ned.ipac.caltech.edu/level5/Guth/Guth3.html (https://ned.ipac.caltech.edu/level5/Guth/Guth3.html)


And then about pressure
https://physics.stackexchange.com/questions/3688/why-does-pressure-act-as-a-source-for-the-gravitational-field (https://physics.stackexchange.com/questions/3688/why-does-pressure-act-as-a-source-for-the-gravitational-field)
Thank you for the appropriate links. Earlier we covered some of Guth’s work, and you see I included his Inflation theory, along with General Relativity in the make up of Big Bang Theory in my reply to Bill S. One key point in your link is about the physics of the false vacuum:

“THE FALSE VACUUM arises naturally in any theory that contains scalar fields … the energy density is minimal not when the field vanishes, but instead at some nonzero value of the field.”

I say this is a key point because the minimal energy density clearly is not “nothingness”, it is just a relative minimum density level of a positive energy density.

Title: Re: What is Nothingness?
Post by: Bogie_smiles on 18/07/2018 21:36:46
Reply #100



If you think of it most of the current knowledge points to a very high energy density in the beginning. The way stars made us is a direct result from transformations needing heat aka 'kinetic motion'. It doesn't really discuss a geometry although we presume there to be needed some sort of confinement for it to exist, well, at least I do :)
I agree with your take on the required “confinement” or energy density containment of extreme density, and I think most people think of the Big Bang as being associated with conditions akin to a super massive black hole of proportions sufficient to contain the hot dense energy ball that BBT/Inflation features.
Quote

But without 'particles' transferring this 'heat' those initial conditions becomes one of pressure, which then links to Einsteins stress energy tensor. It's interesting

And a very high energy density is also about heat.
=
Again you seem to have thought it through, and concluded that unless there were particles present, the containment of the heat seems problematic. You point out that the equivalent pressure may be provided by the EFE stress tenor, and I’m not sharp enough to comment.

However, if the concept of a massive black hole preceding the Big Bang has any merit, such a crunch could be accumulated from galactic matter and stellar/galactic black holes which could feature dense particle containment and heat transfer.
Quote

And if we want to add to that we can consider the homogeneous isotropic universe we find around us. A 'Big Bang' whatever that means is not a centered source, situated at some 'middle of the universe'. In a way it never stopped as every 'point' of this universe still expands. No center to it, and the proof of that lies in astronomical observations of 'early light', being all around us. I can't see any other way to understand it myself.

Great minds have pondered it for a long time, and the consensus BBT/Inflation cosmology is bound to evolve as the scientific community advances our understanding.
Title: Re: What is Nothingness?
Post by: Bill S on 18/07/2018 22:00:02
Quote from: yor_on
There are two very good questions you can ask
What makes a vacuum

Would that be an "absolute" vacuum, which must be nothing; or the sort that fills our Universe, which is obviously something?
Title: Re: What is Nothingness?
Post by: yor_on on 19/07/2018 07:14:52
 Bill " Presumably, you mean the beginning of our "observable" Universe; not the beginning of everything; before which there would have been "nothing"."

It's about 'falsifying'. What it means to me is that it has to be testable, you need to probe it to know. What was 'before' is not yet probe-able, and that places it in meta physics. A pure nothing is still interesting though. If you are a fan of waves one might presume that quenching them, even virtually, might 'produce that sort of of vacuum, but I do think HUP will make problems even then, as well as the second and third law of entropy which state that a zero temperature isn't reachable .

"The Third Law states, “The entropy of a perfect crystal is zero when the temperature of the crystal is equal to absolute zero (0 K).” According to Purdue University, “The crystal must be perfect, or else there will be some inherent disorder. It also must be at 0 K; otherwise there will be thermal motion within the crystal, which leads to disorder.”   

 Siabal Mitra, a professor of physics at Missouri State University, provides another implication of this law. “One version of the Third Law states that it would require an infinite number of steps to reach absolute zero, which means you will never get there. If you could get to absolute zero, it would violate the Second Law, because if you had a heat sink at absolute zero, then you could build a machine that was 100 percent efficient.”   "

And here is a mathematical proof of the third law
https://www.nature.com/articles/ncomms14538

" The Second Law indicates that thermodynamic processes, i.e., processes that involve the transfer or conversion of heat energy, are irreversible because they all result in an increase in entropy. Perhaps one of the most consequential implications of the Second Law, according to Mitra, is that it gives us the thermodynamic arrow of time.

In theory, some interactions, such as collisions of rigid bodies or certain chemical reactions, look the same whether they are run forward or backward. In practice, however, all exchanges of energy are subject to inefficiencies, such as friction and radiative heat loss, which increase the entropy of the system being observed. Therefore, because there is no such thing as a perfectly reversible process, if someone asks what is the direction of time, we can answer with confidence that time always flows in the direction of increasing entropy. "

If that isn't reachable then there must be 'energy' with the possibility of increasing entropy and  HUP is yet another way of looking at it. So I'm not sure how to reach that state? You could also possibly define a vacuum without time, as we both know that time is presumed to be a symmetry of sorts, in physics and mathematics you theoretically can 'wind time backwards', even though there are no proofs of the same existing practically. So stopping time stops fluctuations but how, will, or can, we measure/reach such a state?
Title: Re: What is Nothingness?
Post by: Bill S on 19/07/2018 13:46:38
Quote from: yor_on
It's about 'falsifying'. What it means to me is that it has to be testable, you need to probe it to know.


OK.  If I express the opinion that there can never have been “nothing”; that could be falsified if someone could produce an example of “something from nothing”. Takers are a rarity. :)

Quote
What was 'before' is not yet probe-able, and that places it in meta physics.


True, but surely, we should not simply suspend our logical faculties when considering possibilities.  After all, we are talking about science, not religion. 

Quote
A pure nothing is still interesting though.

So it would seem.
 
Title: Re: What is Nothingness?
Post by: yor_on on 19/07/2018 15:16:42
meta physics isn't suspending ones logical faculties Bill. Both you and me do it, but it's just not as straightforward. One example of it might be to argue that if indeed a vacuum is something, shouldn't there be something it exist in? Both questions are interesting :) but not answerable until someone find that way to probe them by experiments and mathematics. If the universe would be a mathematical space then I don't think it needs anything except a logic f.ex. Which of course is a very large step away from any normal expectation.
Title: Re: What is Nothingness?
Post by: Bill S on 19/07/2018 16:39:26
Quote from: yor_on
meta physics isn't suspending ones logical faculties Bill.

I certainly didn’t intend giving the impression I thought that was the case.  I was making the point that just because something is “not yet probe-able”, we should, perhaps, not treat it as though it were not worthy of logical thought/reasoning.  Your response suggests that our thinking is much the same there.
Title: Re: What is Nothingness?
Post by: Bogie_smiles on 20/07/2018 01:57:54
Reply #106



Elsewhere, I have explained why I would normally use “cosmos” in this context; but “Universe” seems OK.  I would have to include one serious proviso, but this is neither the time nor place to climb into my soap box about eternity/infinity.


Reading back through the thread, I think it is time for Bill S to tell what he meant about preferring to use the word “cosmos” instead of “universe”, and also, why he balked at “climbing on his soap box about eternity/infinity”.
Title: Re: What is Nothingness?
Post by: Bill S on 20/07/2018 22:34:09
Quote from: Bogie Smiles
Reading back through the thread, I think it is time for Bill S to tell what he meant about preferring to use the word “cosmos” instead of “universe”, and also, why he balked at “climbing on his soap box about eternity/infinity”.

It’s not a question of preferring one term to another. As noted above, I’ve explained elsewhere that I generally follow John Gribbin’s suggestion; which I summarised as:

Cosmos = everything that exists, or can exist.
Universe = our (in principle) observable portion of spacetime and its contents.
universe = any other universe that may, or may not, exist.
 
This is a quote from Gribben’s book, “Companion to the Cosmos”. 

    “Universe   With the capital “U”, the term used for everything that we can ever have knowledge of, the entire span of space and time accessible to our instruments, now and in the future.  This may seem like a fairly comprehensive definition, and in the past has traditionally been regarded as synonymous with the entirety of everything that exists.  But the development of ideas such as inflation suggests that there may be something else beyond the boundaries of the observable Universe - regions of space and time that are unobservable in principle, not just because light from them has not yet had time to reach us, or because our telescopes are not sensitive enough to detect their light.  This has led to some ambiguity in the use of the term “Universe”.  Some people restrict it to the observable Universe, while others argue that it should be used to refer to all of space and time.  In this book, we use “Universe” as the name for our own expanding bubble of spacetime, everything that is in principle visible to our telescopes, if we wait long enough for the light to arrive.  We suggest that the term “Cosmos” can be used to refer to the entirety of space and time, within which (if the inflationary scenario is correct) there may be an indefinitely large number of other expanding bubbles of spacetime, other universes with which we can never communicate.”

I hope this clarifies things.

The “soapbox” thing was because I thought it could derail the thread.
Title: Re: What is Nothingness?
Post by: Bogie_smiles on 21/07/2018 00:05:51
Reply #108


It’s not a question of preferring one term to another. As noted above, I’ve explained elsewhere that I generally follow John Gribbin’s suggestion; which I summarised as:

Cosmos = everything that exists, or can exist.
Universe = our (in principle) observable portion of spacetime and its contents.
universe = any other universe that may, or may not, exist.

This is a quote from Gribben’s book, “Companion to the Cosmos”. 

    “Universe   With the capital “U”, the term used for everything that we can ever have knowledge of, the entire span of space and time accessible to our instruments, now and in the future.  This may seem like a fairly comprehensive definition, and in the past has traditionally been regarded as synonymous with the entirety of everything that exists.  But the development of ideas such as inflation suggests that there may be something else beyond the boundaries of the observable Universe - regions of space and time that are unobservable in principle, not just because light from them has not yet had time to reach us, or because our telescopes are not sensitive enough to detect their light.  This has led to some ambiguity in the use of the term “Universe”.  Some people restrict it to the observable Universe, while others argue that it should be used to refer to all of space and time.  In this book, we use “Universe” as the name for our own expanding bubble of spacetime, everything that is in principle visible to our telescopes, if we wait long enough for the light to arrive.  We suggest that the term “Cosmos” can be used to refer to the entirety of space and time, within which (if the inflationary scenario is correct) there may be an indefinitely large number of other expanding bubbles of spacetime, other universes with which we can never communicate.”

I hope this clarifies things.

It does. Gribbin really nails the situation in a way that makes a lot of sense and allows for making some distinctions for various scenarios. In the quote you provided from Gribben’s book, “Companion to the Cosmos”, the last sentence typifies its universal applicability when it refers to the cosmos as “the entirety of space and time, within which there may be an indefinitely large number of other expanding bubbles of spacetime, and other universes with which we can never communicate.”

We have to appreciate how all encompassing that makes the word Cosmos in regard to its applicability to any and all of the various possible models of cosmology.

It can apply to our observable universe and to every reputable model of cosmology that has to be consistent with what we can and do observe. That means it satisfies Big Bang Theory with Inflation Theory, which includes General Relativity, Spacetime, and the expanding universe, and accelerating expansion for that matter. It covers any model that invokes the Cosmological Principle, and it accommodates the cyclical models too. It accommodates Guth’s Inflation Theory with the false vacuum, and it accommodates Quantum Mechanics with all of its Interpretations, meaning it works for Quantum Physics, including Quantum Field Theory and the nucleating bubbles of the false vacuum, and Quantum Chromodynamics, and any QM associated model. It even satisfies the requirements of String Theory with its infinite multiple universes and dimensions. And not the least of which, it applies to any version of a Steady State Theory which go beyond those that invoke the cosmological principle and vacuum energy density to also invoke the Perfect Cosmological Principle that specifies that the universe is homogeneous and isotropic, not only in space, but in space and time, as mentioned in the OP.

Still, I like the versatility of the definition offered in this thread which has the same range of applicability as Gribbin’s full scope definition, but that also has something that I am going for that might not have been necessary to Gribbin in the context of his book. In the context of this thread that starts out about “nothingness” I was interested in making “universe” the antithesis of “nothingness”, and so universe is everything that nothingness is not.

Nothingness: No space, no time, no energy, and no potential for any space, time or energy.
Universe: There is just one universe and it encompasses all there is, all matter, energy, everything, in one infinite and eternal presence, that had no beginning and will have no end, i.e., the universe has always existed, and has always been governed by the same set of invariant natural laws.


Edit Note: Thank you for your participation in this thread. I have linked the content of it to my on-going thread in the “On the Lighter Side” forum category, in the New Theories Sub-forum and to my thread, “If there was one Big Bang event, why not multiple Big Bang events”: https://www.thenakedscientists.com/forum/index.php?topic=70348.msg548939#msg548939 (https://www.thenakedscientists.com/forum/index.php?topic=70348.msg548939#msg548939)

Title: Re: What is Nothingness?
Post by: Zer0 on 22/07/2018 20:27:24
.
Title: Re: What is Nothingness?
Post by: Bill S on 24/07/2018 20:56:27
I'm mystified by #109.  Is it a link to facebook?
Title: Re: What is Nothingness?
Post by: Bogie_smiles on 24/07/2018 21:21:25
I'm mystified by #109.  Is it a link to facebook?
Actually Zero was being clever. The post is close to nothingness, but does consist of a single period in a light color, maybe white or grey.
Title: Re: What is Nothingness?
Post by: Bill S on 24/07/2018 23:58:24
Too smart for me. :)

I put it in my search bar and ended up here:
https://www.facebook.com/ArtCtrlDel/posts/%221n73ll1g3nc3-15-7h3-4b1l17y-70/1001986609936346/

Database Error

Please try again. If you come back to this error screen, report the error to an administrator.
Back